You are on page 1of 111

Total Marks : 200

TEST - 1 (MOCK TEST - 1) Mark Scored : 103.33

1 Social Entrepreneurship is running

A. Any non-profit business


B. An ecologically sustainable business
C. An inclusive business that has participation from major social groups and local community
D. A business that makes its money in a socially responsible way

Your Answer : D
Correct Answer : D

Answer Justification :

Learning: It is the use of the techniques by start-up companies and other entrepreneurs to develop,
fund and implement solutions to social, cultural, or environmental issues.

Muhammad Yunus (Grameen Bank) was a famous social entrepreneur. Another example can be a
crowdfunding internet platform like https://milaap.org 77 2
2
These ventures are not necessarily formed to reinvest all profits into the5communities. 03
4 1
-9
So, these firms apply commercial strategies to maximize improvements in human and
m
environmental well-being.
il .co
a
mthe PM
Q Source: Often seen in news: A recent speechg by
a 7@
is r
it m
l k
- pu
i sra
k itM
l
Pu

www.insightsias.com 1
© Insights Active Learning | All rights reserved - 78168. You may not reproduce, distribute or exploit the contents in any form without
written permission by copyright owner. Copyright infringers may face civil and criminal liability
Total Marks : 200
TEST - 1 (MOCK TEST - 1) Mark Scored : 103.33

2 Which of these are the chief difference(s) between Mesolithic and Neolithic cultures?
1. Hunting-gathering pattern of life began not before the Neolithic period.
2. Mesolithic cultures did not practice domestication of animals unlike Neolithic people.
3. The use of bow and arrow cannot be found in Mesolithic culture, unlike Neolithic culture.
4. Evidence of pottery is absent from the Neolithic culture but is markedly found in Mesolithic culture.

Select the correct answer using the codes below.


A. 3 and 4 only
B. 2 only
C. 1 and 3 only
D. None of the above

Your Answer : C
Correct Answer : D

Answer Justification : 7
2 27
5 03
Justification: Statement 1 and 3: Hunting-gathering pattern of life was prevalent since Old stone
age to Mesolithic and Neolithic.
9 41
-
However, there seems to have been a shift from big animal hunting
o m to small animal hunting and
. c
fishing. This is how bows and arrows were used in thisilperiod. Both 1 and 3 are thus wrong.
m a
g
7@
Statement 2: Domestication of animals, horticulture and primitive cultivation started during
Mesolithic period itself.
sr a
m i
However, during Neolithic period, k i tdomestication of sheep, goats and cattle was widely prevalent.
u l
p and for transport.
Cattle were used for cultivation
-
a
srNeolithic
Statement 4: During
M i age, wheels were used to make pottery. Pottery was used for cooking

l k it of food grains. So, 4 is wrong.


as well as storage

Pu
Learning: During the Neolithic phase, the cultivation of plants and domestication of animals led to
the emergence of village communities based on sedentary life.

There was a great improvement in technology of making tools and other equipments used by
man. Stone tools were now polished.

Mud brick houses were built instead of grass huts.

Large urns were used as coffins for the burial of the dead.

Wheat, barely, rice, millets were cultivated in different areas at different points of time. Rice
cultivation was extensive in eastern India.

The people of Neolithic Age used clothes made of cotton and wool
www.insightsias.com 2
© Insights Active Learning | All rights reserved - 78168. You may not reproduce, distribute or exploit the contents in any form without
written permission by copyright owner. Copyright infringers may face civil and criminal liability
Total Marks : 200
TEST - 1 (MOCK TEST - 1) Mark Scored : 103.33

Q Source: Page 13-14: TN 11th Standard History Textbook

7
2 27
0 3
5
41
-9
. com
ai l
gm
7 @
isra
i t m
p ulk
a -
i sr
M
l k it
Pu

www.insightsias.com 3
© Insights Active Learning | All rights reserved - 78168. You may not reproduce, distribute or exploit the contents in any form without
written permission by copyright owner. Copyright infringers may face civil and criminal liability
Total Marks : 200
TEST - 1 (MOCK TEST - 1) Mark Scored : 103.33

3 The Government of India has recently announced the sale of “Government of India Floating Rate Bonds
2024”. With reference to government securities, consider the following:
1. Floating Rate Bonds are sold by the Reserve Bank of India.
2. RBI issues Cash Management Bills (CMBs) to meet long-term borrowing needs of the Government
of India.
3. Treasury bills are short term debt instruments issued by the Government of India.

Select the correct answer using the codes below.


A. 1 and 2 only
B. 1 and 3 only
C. 2 only
D. 2 and 3 only

Your Answer : B
Correct Answer : B

7
Answer Justification :
2 27
0 3
15Central Government or the
Concept: A Government security is a tradable instrument issued by the
4
State Governments. -9
. com
It acknowledges the Government’s debt obligation
l
ai (to raise money from the market for
m
various purposes).
7 @g
isra
Such securities are short term
k i tm(usually called treasury bills, with original maturities of less
than one year) or longu l (usually called Government bonds or dated securities with
term
p
original maturity of- one year or more).
i sra
M
t providing
l k
Besides i a return in the form of coupons (interest), Government securities offer the
u
Pmaximum safety as they carry the Sovereign’s commitment for payment of interest and
repayment of principal.

Justification: Statement 1: There are two types of bonds:

Fixed Rate Bonds – These are bonds on which the coupon rate is fixed for the entire life of the
bond. Most Government bonds are issued as fixed rate bonds.

Floating Rate Bonds – Floating Rate Bonds are securities which do not have a fixed coupon rate.
The coupon is re-set at pre-announced intervals (say, every six months or one year) by adding a
spread over a base rate. These are issued by RBI.

Statement 2: Cash Management Bills (CMBs), are a relatively new short-term instrument to meet
the temporary mismatches in the cash flow of the Government. The CMBs have the generic
character of T-bills but are issued for maturities less than 91 days.

Statement 3: Treasury bills or T-bills, which are money market instruments, are short term debt
instruments issued by the Government of India and are presently issued in three tenors, namely, 91
www.insightsias.com 4
day, 182 day and 364 day.
© Insights Active Learning | All rights reserved - 78168. You may not reproduce, distribute or exploit the contents in any form without
written permission by copyright owner. Copyright infringers may face civil and criminal liability
Total Marks : 200
TEST - 1 (MOCK TEST - 1) Mark Scored : 103.33

Q Source: RBI Bond sales

7
2 27
0 3
5
41
-9
. com
ai l
gm
7 @
isra
i t m
p ulk
a -
i sr
M
l k it
Pu

www.insightsias.com 5
© Insights Active Learning | All rights reserved - 78168. You may not reproduce, distribute or exploit the contents in any form without
written permission by copyright owner. Copyright infringers may face civil and criminal liability
Total Marks : 200
TEST - 1 (MOCK TEST - 1) Mark Scored : 103.33

4 Ashokan inscriptions remain valuable sources for the study of Asoka and the Mauryan Empire. This is
because
1. His wooden pillars heralded a new architectural era because most of the monuments before his
period were made of stone.
2. Pillar edicts give a summary of his efforts to promote the Dhamma within his kingdom and outside.
3. They often deal with instructions given to his officials which is informative of Mauryan polity.

Select the correct answer using the codes below.


A. 2 and 3 only
B. 1 and 3 only
C. 1 and 2 only
D. 2 only

Your Answer : A
Correct Answer : A

7
Answer Justification :
2 27
0 3
15were mostly made of wood
Justification: Statement 1: The monuments before the period of Asoka
4
and therefore perished. -9
om
c numerous monuments of Asoka, only a
The use of stone started from the time of Asoka. Even of
a i l.the
few have remained. m
7 @g
s a
His palace and monasteries and most of r his stupas have disappeared. The only remaining stupa is at
Sanchi. i
k i tm
ul of Dhamma gives crucial information about the social life at the
Statement 2: The state andpspread
time of Ashoka. a -
is r
M
Statement 3:itAshoka often gave instructions to his Dhamma mahamahattas to spread Dhamma.
u lk
These instructions, their style etc were recorded on the edicts.
P
Learning: Ashokan inscriptions (first deciphered by James Princep) are written in Pali language
and in some places Prakrit was used.

The Brahmi script was employed for writing. In the northwestern India Asokan inscriptions were
found in Karoshti script.

There are fourteen Major Rock Edicts. The two Kaling Edicts are found in the newly conquered
territory. The major pillar Edicts were erected in important cities. There are minor Rock Edicts and
minor pillar Edicts.

Q Source: Page 60-61: TN 11th Standard History Textbook

www.insightsias.com 6
© Insights Active Learning | All rights reserved - 78168. You may not reproduce, distribute or exploit the contents in any form without
written permission by copyright owner. Copyright infringers may face civil and criminal liability
Total Marks : 200
TEST - 1 (MOCK TEST - 1) Mark Scored : 103.33

5 Which of the following is the closest analogy of the Indian Constitution?

A. Rulebook
B. Anthology
C. Dictionary
D. Social Register

Your Answer : A
Correct Answer : A

Answer Justification :

Justification: Option A: The Indian Constitution lays down the basic rules or laws that have to be
followed by everyone. These laws are for both the government and the people. Hence, A is
appropriate.

77constitution is
Option B: An anthology is a collection of literary works, originally of poems, which
2
not. 3 2
1 50
94terms, it is much more than
Option C: Constitution is not a mere dictionary of political and legal
-
om
that.
l . c
aifrom certain social classes. Constitution is for
Option D: Social Register (SR) is a directory of people
m
everyone. It isn’t a directory.
7 @g
Q Source: Improvisation: Page 39: NCERT isra Class VI Social and Political Life - I
k i tm
p ul
a -
is r
M
l k it
Pu

www.insightsias.com 7
© Insights Active Learning | All rights reserved - 78168. You may not reproduce, distribute or exploit the contents in any form without
written permission by copyright owner. Copyright infringers may face civil and criminal liability
Total Marks : 200
TEST - 1 (MOCK TEST - 1) Mark Scored : 103.33

6 ‘JIGYASA’, recently seen in news is a

A. Interstellar exploration programme of ISRO


B. Student- scientist connect programme
C. Earth core drilling programme of Ministry of Earth Sciences
D. Biodiversity documentation programme in the Western Ghats

Your Answer : B
Correct Answer : B

Answer Justification :

Learning: Council of Scientific and Industrial Research (CSIR), has joined hands with Kendriya
Vidyalaya Sangathan (KVS) to implement this programme.

The program will also enable the students and teachers to practically live the theoretical concepts
77projects. The
taught in science by visiting CSIR laboratories and by participating in mini-science
2
model of engagement includes: 3 2
5 0
41
Student Residential Programmes; -9
. com
ai l
gm
Scientists as Teachers and Teachers as Scientists;
a 7@
isr
tmExperiments;
Lab specific activities / Onsite
i
lk
- pu
i sra school students and scientists so as to extend student’s classroom
The focus is on connecting
learning with that of a very well-planned research laboratory based learning.
k itM
l
Pu http://pib.nic.in/newsite/PrintRelease.aspx?relid=167194
Q Source:

www.insightsias.com 8
© Insights Active Learning | All rights reserved - 78168. You may not reproduce, distribute or exploit the contents in any form without
written permission by copyright owner. Copyright infringers may face civil and criminal liability
Total Marks : 200
TEST - 1 (MOCK TEST - 1) Mark Scored : 103.33

7 Consider the following statements. Assertion (A): Every elector at the Presidential election is bound by
the voting instructions issued by the whip of the respective political party. Reason (R): Anti-defection
provisions are enforced by the constitution under the Tenth Schedule.

In the context of the above, which of these is correct?


A. A is correct, and R is an appropriate explanation of A.
B. A is correct, but R is not an appropriate explanation of A.
C. A is correct, but R is incorrect.
D. A is incorrect, but R is correct.

Your Answer : D
Correct Answer : D

Answer Justification :

Concept: Defection is defined as “to abandon a position or association, often to join an opposing
group” which essentially describes a situation when a member of a particular party
277abandons his
2
3otherwise) to another
loyalty towards that party and provide his support (in the form of his vote or
5 0
party. 1
- 94
m
th
To avoid such a situation (about which you’ll read later in Laxmikanth and 11 NCERT), an anti-
co
a i l.
defection provision was added in the constitution. If a member of a political party disobeys party
directions concerning matters such as voting on a bill or for a candidate, this amounts to defection.
g m
7@
The member is then liable to be punished by the party.
r a
m is
Justification: However, the ECI has clarified recently that voting or not voting as per his/her own
t
ki and the electors are at liberty to vote or not to vote at the
free will at the Presidential election
u l
Presidential election as perptheir own free will and choice.
a -
is r
This willt M
equally apply to the political parties and they are free to canvas or seek votes of
k i
l for any candidate or requesting or appealing to them to refrain from voting.
Pu
electors

However, the political parties cannot issue any direction or whip to their members to vote in a
particular manner or not to vote at the election leaving them with no choice, as that would
tantamount to the offence of undue influence within the meaning of section 171C of the IPC.

This is because, as per the ECI, voting at election to the office of President is different from
voting by a member of Parliament or State Legislature inside the House.

Q Source: RM/ RS: Release ID :167193

www.insightsias.com 9
© Insights Active Learning | All rights reserved - 78168. You may not reproduce, distribute or exploit the contents in any form without
written permission by copyright owner. Copyright infringers may face civil and criminal liability
Total Marks : 200
TEST - 1 (MOCK TEST - 1) Mark Scored : 103.33

8 With reference to unique BRICS countries initiatives, consider the following statements:
1. BRICS Network University (NU) is an open educational project and does not involve establishment
of rigid organizational forms such as a secretariat.
2. BRICS Think Tanks Council (BTTC) comprises research organizations from BRICS countries
including Observer Research Foundation (ORF) from India.

Which of the above is/are correct?


A. 1 only
B. 2 only
C. Both 1 and 2
D. None

Your Answer : B
Correct Answer : C

Answer Justification : 7
2 27
03 declaration on
th
Justification: Recently at the 5 BRICS Education ministers meeting, Beijing
5
41
education was adopted which advocated support for both these organizations.
9
-
o m
Statement 1: NU is a consortium of the universities, elaborating Master and PhD programmes in
c
il. meeting at Moscow.
different disciplines. It was established in 2015 in BRICS
m a
g
NU follows principles of:
a 7@
is r
m
it universities to join the network respecting national criteria.
Openness, which enableskvarious
l
- pu
i sra programmes that can be supplemented with network research and
Focus on educational

k it Mprojects.
innovation
l
Pu
There is also a BRICS Universities League which is a consortium of leading research universities
from BRICS countries.

Statement 2: The BTTC (established in 2013) comprises the Institute for Applied Economic
Research (IPEA) (Brazil), the National Committee for BRICS Research (NRC/BRICS) (Russia), the
Observer Research Foundation (ORF) etc.

The main purposes of its establishment was to form a platform for the exchange of ideas and
present policy recommendations for BRICS leaders.

Q Source: Beijing declaration on Education: 5th July 2017:


http://pib.nic.in/newsite/PrintRelease.aspx?relid=167154

www.insightsias.com 10
© Insights Active Learning | All rights reserved - 78168. You may not reproduce, distribute or exploit the contents in any form without
written permission by copyright owner. Copyright infringers may face civil and criminal liability
Total Marks : 200
TEST - 1 (MOCK TEST - 1) Mark Scored : 103.33

9 With reference to the Sangam age, Panar and Viraliyar were

A. Temple guards
B. Bull fighters
C. Singing bards
D. Charity donors

Your Answer : C
Correct Answer : C

Answer Justification :

Learning: Poetry, music and dancing were popular among the people of the Sangam age. Liberal
donations were given to poets by the kings, chieftains and nobles.

77They were
The royal courts were crowded with singing bards called Panar and Viraliyar.
2
experts in folk songs and folk dances. 3 2
5 0
9 41
-
The arts of music and dancing were highly developed. A variety of Yazhs and drums are
m
referred to in the Sangam literature. co il .
m a
@ g
Dancing was performed by Kanigaiyar.
a 7 Koothu was the most popular entertainment of the
people. s r i
m
ul kit
Q Source: Page 91: TN -11p
th
Standard History Textbook

i sra
M
l k it
Pu

www.insightsias.com 11
© Insights Active Learning | All rights reserved - 78168. You may not reproduce, distribute or exploit the contents in any form without
written permission by copyright owner. Copyright infringers may face civil and criminal liability
Total Marks : 200
TEST - 1 (MOCK TEST - 1) Mark Scored : 103.33

10 The 600-year-old Walled City of Ahmedabad was recently declared a World Heritage City. With
reference to the World Heritage Cities Programme and associated initiatives, consider the following:
1. It is a thematic programmes formally approved and monitored by the World Heritage Committee.
2. The programme has provisions for technical assistance to States Parties for the implementation of
new approaches and schemes for urban heritage conservation.
3. IUCN has set up the ‘Historic Urban Landscape initiative’ which is an international working group
comprising, inter alia, UNESCO as an advisory body to the World Heritage Convention.
4. The Organization of World Heritage Cities (OWHC) is an inter-governmental organization of the
municipal corporations of cities in which sites of the UNESCO World Heritage list are located.

Select the correct answer using the codes below.


A. 1 and 2 only
B. 3 and 4 only
C. 1, 2 and 3 only
D. 1, 2, 3 and 4

7
Your Answer :
2 27
Correct Answer : A 0 3
5
41
Answer Justification : -9
om
il.cParties in the challenges of protecting and
Justification: Statement 1 and 2: It aims to assist States
a
managing their urban heritage. The programme ism structured along a two-way process, with 1) the
@
development of a theoretical framework for urban
g heritage conservation, and 2) the provision of
a 7
srthe implementation of new approaches and schemes.
technical assistance to States Parties for
i
t m
lki process of conservation, UNESCO (not IUCN) set up the Historic
Statement 3: As part of this policy
u
Urban Landscape initiative,
- p an international working group comprising ICOMOS, IUCN and
ICCROM (as Advisory
i sraBodies to the 1972 World Heritage Convention) and other partner
t M
organizations, including UN-Habitat etc.
i
lk4: OWHC is an international non-profit, non-governmental organization of 250 cities in
P u
Statement
which sites of the UNESCO World Heritage list are located.

Learning: It is first Indian city get the honour. The Walled City of Ahmedabad is located on the
eastern banks of Sabarmati river.

It was founded by Sultan Ahmed Shah I of Gujarat Sultanate in in 1411.

It remained the capital of the Gujarat Sultanate and later important political and commercial
centre of Gujarat.

It presents a rich architectural heritage from the sultanate period, notably the Bhadra citadel,
walls and gates of the Fort city and numerous mosques and tombs, as well as important Hindu
and Jain temples of later periods.

Q Source: Improvisation:
www.insightsias.com 12
http://www.thehindu.com/news/national/other-states/ahmedabad-declared-indias-first-heritage-city-
© Insights Active Learning | All rights reserved - 78168. You may not reproduce, distribute or exploit the contents in any form without
written permission by copyright owner. Copyright infringers may face civil and criminal liability
Total Marks : 200
TEST - 1 (MOCK TEST - 1) Mark Scored : 103.33

by-unesco/article19244478.ece

7
2 27
0 3
5
41
-9
. com
ai l
gm
7 @
isra
i t m
p ulk
a -
i sr
M
l k it
Pu

www.insightsias.com 13
© Insights Active Learning | All rights reserved - 78168. You may not reproduce, distribute or exploit the contents in any form without
written permission by copyright owner. Copyright infringers may face civil and criminal liability
Total Marks : 200
TEST - 1 (MOCK TEST - 1) Mark Scored : 103.33

11 What argument(s) would you advance to justify that the position of women had become miserable
during the Gupta period?
1. Women were prohibited from studying religious texts.
2. The practice of Swyamvara was given up and Manusmriti suggested early marriage for girls.

Which of the above is/are correct?


A. 1 only
B. 2 only
C. Both 1 and 2
D. None

Your Answer : A
Correct Answer : C

Answer Justification :
7
Justification: Statement 1: This ensured the domination of men over women.
2 27
0 3
Statement 2: Swaymvara was the practice of women choosing their own
4 15partners by way of a
- 9
competition/selection. When this was given up, women lost an important right. Manusmriti (law
m of women.
book) further entrenched patriarchal rules to worsen the position
i l .co
Learning: During the Gupta dynasty period the maleamembers of a family were entitled to inherit
gm right of inheritance was not given to women or
7@
property left behind by their fathers. However, such
girls. r a
m is
The power of inheritance enabled
l k itthe male members of a family to exercise dominance over the
women.
- pu
i
Q Source: Page 104: sraTN 11 Standard History Textbook
th

k itM
l
Pu

www.insightsias.com 14
© Insights Active Learning | All rights reserved - 78168. You may not reproduce, distribute or exploit the contents in any form without
written permission by copyright owner. Copyright infringers may face civil and criminal liability
Total Marks : 200
TEST - 1 (MOCK TEST - 1) Mark Scored : 103.33

12 The year of 2017 marks 25 years of dialogue partnership between India and ASEAN and 15 years of
summit-level interaction. With reference to India-ASEAN relations, consider the following:
1. India's relationship with ASEAN is a key pillar of our Act East Policy.
2. AIFTA is a free trade area among the member states of the ASEAN and India.
3. India and ASEAN are yet to upgrade their relationship to a ‘strategic partnership’.

Select the correct answer using the codes below.


A. 2 only
B. 1 and 3 only
C. 1 and 2 only
D. 3 only

Your Answer : B
Correct Answer : C

Answer Justification :
7
2 27
Justification: Statement 1: India’s commitment to strengthen its ties with 3 ASEAN countries is
50 the focus -Commerce,
reflected by its ‘Look East Policy’ becoming ‘Act East Policy.’ Three Cs1are
4
Connectivity and Culture. -9
. comcountries: Australia, New Zealand and
Statement 2: ASEAN became ASEAN Plus Six with additional l
ai has seen progress through the
India. Codification of the relations between these mnations
development of the Regional Comprehensive @ g
Economic Partnership (RCEP), a proposed free-trade
agreement involving the 16 countries of r a 7
is ASEAN plus six.
Statement 3: India and ASEANlk itm
had upgraded their relationship to a strategic partnership in 2012.
p u
a
ASEAN consists of 10-member- countries- Brunei, Cambodia, Indonesia, Laos, Malaysia, Myanmar,
i sr
Philippines, Singapore, Thailand and Vietnam.
k itM
P ul
It is a regional organisation that aims to promote intergovernmental cooperation and economic
integration amongst its members.

Q Source: 25 years of Dialogue partnership between India-ASEAN

www.insightsias.com 15
© Insights Active Learning | All rights reserved - 78168. You may not reproduce, distribute or exploit the contents in any form without
written permission by copyright owner. Copyright infringers may face civil and criminal liability
Total Marks : 200
TEST - 1 (MOCK TEST - 1) Mark Scored : 103.33

13 Universal suffrage (voting rights to all eligible) was granted to women all over India in

A. 1950 once Indian republic was formed


B. Morley-Minto Act
C. Government of India Act, 1935
D. 1939 General elections

Your Answer : A
Correct Answer : A

Answer Justification :

Justification & Learning: First local effort was made in Madras, which granted women’s suffrage
in 1921, but only to those men and women who owned land property according to British
administration's records. This was neither Universal nor equitable.

In 1950, universal suffrage granted voting rights to all women. This is enshrined7in7Article 326 in
2 2
our constitution. 3 0
5
9 Life - I
Q Source: Improvisation: Page 32: NCERT Class VI Social and Political 41
-
. com
a i l
m
7 @g
isra
it m
p ulk
a -
i sr
M
l k it
Pu

www.insightsias.com 16
© Insights Active Learning | All rights reserved - 78168. You may not reproduce, distribute or exploit the contents in any form without
written permission by copyright owner. Copyright infringers may face civil and criminal liability
Total Marks : 200
TEST - 1 (MOCK TEST - 1) Mark Scored : 103.33

14 Karla is one of the most famous centres of early rock-cut architecture and has received much
attention of scholars and commoners alike. What is important about Karla caves?
1. The grand Chaitya griha of Karla is the largest of all the chaita-grihas of India.
2. The rock-cut cave was completed without accepting any financial donation from individuals apart
from their voluntary labour.

Which of the above is/are correct?


A. 1 only
B. 2 only
C. Both 1 and 2
D. None

Your Answer :
Correct Answer : A

Answer Justification :
7
2 27
3 in the whole of India.
Justification: Statement 1: The chaityagriha at Karla is the biggest of its type
0
The hall measures around thirty eight m deep from door to back. The 1 5
object of worship is the stupa
9 4
at the rear end of the chaityagriha. -
m
co support of a group of assorted
Statement 2: The caves were caused from the donationsl.and
i
m a merchants, monks etc.
individuals. This includes a prince of Maharathi family;

7 @g
sra
The inscriptions of Usavadata and Vasisthiputra Pulumavi mention donation of land to the Veluraka
Samgha for this. i
k i tm
l VI Our Pasts - I
uClass
Q Source: Page 105: NCERT
- p
r a
M is
l k it
Pu

www.insightsias.com 17
© Insights Active Learning | All rights reserved - 78168. You may not reproduce, distribute or exploit the contents in any form without
written permission by copyright owner. Copyright infringers may face civil and criminal liability
Total Marks : 200
TEST - 1 (MOCK TEST - 1) Mark Scored : 103.33

15 With reference to the National Disaster Response Force (NDRF), consider the following statements:
1. President of India serves as the Director General (DG) of NDRF.
2. NDRF was established under the Disaster Management Act, 2005.
3. It is housed directly under the Prime Minister’s Office and functions from there.

Select the correct answer using the codes below.


A. 2 only
B. 1 and 3 only
C. 1 only
D. 2 and 3 only

Your Answer : A
Correct Answer : A

Answer Justification :
7
27under Disaster
Justification: NDRF is India’s elite disaster mitigation combat force established
2
Management Act, 2005 3 0
5
41
- 9 as Director General (DG) of
Statement 1: Senior IPS officer Sanjay Kumar was recently appointed
National Disaster Response Force (NDRF). m
i l .co
Statement 3: It functions under Union Ministry of Home
m a Affairs. It is mandated to undertake special
disaster response, relief, rescue operations and
7 @gcombat roles independently in the case of an event
sra or emergency.
of any disaster (natural or man-made), accident
i
k
It also assists local authorities in i tm
launching a quick rescue and response operation to save life and
u l
property.
- p
Q Source: i sra
itM
http://www.thehindu.com/news/national/andhra-pradesh/ndrf-ready-to-tackle-emergencies-in-monso
k
l
Pu
on-says-its-commandant/article18958062.ece

www.insightsias.com 18
© Insights Active Learning | All rights reserved - 78168. You may not reproduce, distribute or exploit the contents in any form without
written permission by copyright owner. Copyright infringers may face civil and criminal liability
Total Marks : 200
TEST - 1 (MOCK TEST - 1) Mark Scored : 103.33

16 With reference to Ancient India, Ashtangasamgraha written by Vagbhata deals with

A. Astrology
B. Medicine
C. Law
D. Economics

Your Answer : B
Correct Answer : B

Answer Justification :

Learning: In the field of medicine, Vagbhata lived during this period. He was the last of the great
medical trio of ancient India.

77 was the
The other two scholars Charaka and Susruta lived before the Gupta age. Vagbhata
2
3
author Ashtangasamgraha (Summary of the eight branches of medicine). 2
5 0
9 41
- with Sushruta Samhita providing
The Sushruta and Charaka texts differ in one major aspect,
m
co primarily a foundation of medicine.
the foundation of surgery, while Charaka Samhita being
il .
m a
@ g
7 Textbook
Q Source: Page 109: TN 11th Standard History
isra
it m
p ulk
a -
i sr
M
l k it
Pu

www.insightsias.com 19
© Insights Active Learning | All rights reserved - 78168. You may not reproduce, distribute or exploit the contents in any form without
written permission by copyright owner. Copyright infringers may face civil and criminal liability
Total Marks : 200
TEST - 1 (MOCK TEST - 1) Mark Scored : 103.33

17 Which of these is/are mandatory qualification(s) to contest for the office of Vice-President of India?
1. She must be a citizen of India or a person of Indian Origin (PIO) residing in India since last ten
years.
2. She must have completed 35 years of age.
3. She must not hold an office for profit.
4. She must have served as a Member of Parliament (MP) or a member of Legislative Assembly in
India.

Select the correct answer using the codes below.


A. 2 and 3 only
B. 2, 3 and 4 only
C. 1 and 4 only
D. 3 only

Your Answer : A
Correct Answer : A
7
2 27
Answer Justification : 0 3
5
41
- 9 of Vice-President of India
Justification: As per article 66, the candidate contesting for election
should be citizen of India, completed age of 35 years and must
. combe qualified to become a member of
Rajya Sabha. He must not hold an office for profit.
a i l
m
7 @g
Her main function is in the form of ex-officio Chairman of Rajya Sabha. She works as acting
sr
president when president is not available. a
i
k i tm Vice Presidential election is also indirect and is held via
ul
Learning: Like Presidential election,
- p
system of proportional representation by means of the single transferable vote.

i sra
it M college of Vice President includes elected by elected members of Lok Sabha and
The electoral
k
ul Sabha.
PRajya
Unlike in Presidential election, MLAs or MLCs from state assemblies have no role to play in
Vice Presidential election.

We’ll explore this better when covering Laxmikanth.

Q Source: ECI has issued a notification for the process of filing nominations for the vice-
presidential election - Incumbent Vice President Hamid Ansari’s term comes to an end on August
10, 2017.

www.insightsias.com 20
© Insights Active Learning | All rights reserved - 78168. You may not reproduce, distribute or exploit the contents in any form without
written permission by copyright owner. Copyright infringers may face civil and criminal liability
Total Marks : 200
TEST - 1 (MOCK TEST - 1) Mark Scored : 103.33

18 Under the 2007 India-Bhutan Friendship Treaty


1. Neither Government will allow the use of its territory for activities harmful to the national security
and interest of the other
2. Both governments allow free trade and commerce between the territories of Bhutan and India.
3. Bhutanese subjects residing in Indian territories shall have equal justice with Indian subjects.
4. Government of Bhutan is free to export those arms, for commercial purposes, that it received via
imports through the territory of India.

Select the correct answer using the codes below.


A. 2 and 4 only
B. 1, 2 and 3 only
C. 1 and 3 only
D. 1, 2, 3 and 4

Your Answer : C
Correct Answer : B
7
2 27
Answer Justification : 0 3
5
41
9 construction party had
Background: In June 2017, Chinese Army (People’s Liberation -Army)
entered the Doklam area and attempted to construct a road.
. com
Royal Bhutan Army (RBA) patrol had
i l
attempted to dissuade them from this unilateral activity.
a
m
7 @g
Since these developments, India and
is raBhutan have been in continuous contact, giving a view of
m and matters of mutual interest.
India’s role in Bhutan’s security
it
p ulk
a - between Indian and Chinese troops in Doklam (located at the tri-junction
The present standoff
r
is and Bhutan) has highlighted India’s very special relationship with Bhutan,
of India, China
M
it military responsibilities.
including
lk
P u
Justification: The 2007 treaty is meant to respect the sensitivities of Bhutan regarding its
sovereignty.

Statement 1: This is to avoid Chinese intrusion from the soil of Bhutan.

Statement 2 and 3: You can read the treaty (hardly 2 pages) here
https://mea.gov.in/Images/pdf/india-bhutan-treaty-07.pdf

Statement 4: Government of Bhutan shall be free to import, from or through India into Bhutan,
whatever arms, ammunition etc as may be required for Bhutan, and that this arrangement shall
hold good for all time as long as the Government of India is satisfied that the intentions of the
Government of Bhutan are friendly and that there is no danger to India from such importations.

However, there shall be no export of such arms, ammunition and materials outside Bhutan either by
the Government of Bhutan or by private individuals. So, 4 is wrong.

Q Source: As mentioned in the background


www.insightsias.com 21
© Insights Active Learning | All rights reserved - 78168. You may not reproduce, distribute or exploit the contents in any form without
written permission by copyright owner. Copyright infringers may face civil and criminal liability
Total Marks : 200
TEST - 1 (MOCK TEST - 1) Mark Scored : 103.33

19 Sunderbans delta is facing potential threats due to


1. Global warming induced sea level rise leading to coastal erosion
2. Freshwater flows from Brahmaputra and Meghna

Which of the above is/are correct?


A. 1 only
B. 2 only
C. Both 1 and 2
D. None

Your Answer : A
Correct Answer : A

Answer Justification :

Justification: Sundarben delta hosts a large population of mangorves (that grow in salt water) - the
7
largest tidal halophytic mangrove forest in the world. 27
0 32
4 15
Statement 1: A study highlights a time series of the erosion of many mangrove forested islands of
the Indian Sunderbans from 1986 to 2012.
- 9
m
i l .co
a Dalhousie, Bhangaduni and Jambudwip.
These islands include Sagar, Gosaba, Dulibhasani,
m
7 @g
isra a driving factor for coastal erosion, coastal flooding,
The mean sea level rise is considered
i tmcreeks and losing land.
increase in the number of tidal
k
p ul
a -
i sr
Statement 2: A critical minimal inflow of freshwater is necessary for the luxuriant growth of
mangroves.
k itM
l
Pu in freshwater inflow results change in mangrove succession where freshwater loving
Decrease
species of mangroves are replaced by salt-water loving ones. So, 2 is wrong.

The loss land, including mangrove forest is mainly due to decrease in fresh water flow and
sediment supply in the western (Indian) part of the delta, and the rate of sea level rise is
higher than sediment supply.

However, the eastern (Bangladesh) side of the Sunderbans delta is gaining land because of
the huge amount of sediment and water flow from the Brahmaputra and Meghna rivers
(which is good for the Mangroves).

This immediate impact of salinity will also have negative impact on the fishing community, as
commercially sought fish species will be replaced by fish that does not have as much market
value.

Q Source:
www.insightsias.com 22
© Insights Active Learning | All rights reserved - 78168. You may not reproduce, distribute or exploit the contents in any form without
written permission by copyright owner. Copyright infringers may face civil and criminal liability
Total Marks : 200
TEST - 1 (MOCK TEST - 1) Mark Scored : 103.33

http://www.thehindu.com/sci-tech/energy-and-environment/climate-change-impact-sunderbans-stea
dily-losing-its-famed-mangroves/article19195229.ece

7
2 27
0 3
5
41
-9
. com
ai l
gm
7 @
isra
i t m
p ulk
a -
i sr
M
l k it
Pu

www.insightsias.com 23
© Insights Active Learning | All rights reserved - 78168. You may not reproduce, distribute or exploit the contents in any form without
written permission by copyright owner. Copyright infringers may face civil and criminal liability
Total Marks : 200
TEST - 1 (MOCK TEST - 1) Mark Scored : 103.33

20 What is the usefulness of Double Asteroid Redirection Test (DART) being developed by NASA?
1. It will help protect earth from potential cosmic body impacts in the future.
2. It will lead to the establishment of the first human space station on an asteroid.

Which of the above is/are correct?


A. 1 only
B. 2 only
C. Both 1 and 2
D. None

Your Answer : A
Correct Answer : A

Answer Justification :

Justification: This is the first-ever mission that will deflect a near-Earth asteroid.
7
2 27
0 3
NASA will use kinetic impactor technique to deflect them. 5
41
-9
o
This technique involves sending one or more large,chigh-speed
mspacecraft into the path of an
i l .
approaching near-earth object to shift its orbitato defend against future impact.
gm
@
It works by changing the speed ofsr
a7
i a threatening asteroid by a small fraction of its total
velocity. tm i
lk
- pu
i srabefore the predicted impact so that this small velocity adds up over time to a
This is done well
Mthe asteroid’s path away from Earth
big shift of
l k it
Pu
Q Source: https://www.nasa.gov/planetarydefense/aida/

www.insightsias.com 24
© Insights Active Learning | All rights reserved - 78168. You may not reproduce, distribute or exploit the contents in any form without
written permission by copyright owner. Copyright infringers may face civil and criminal liability
Total Marks : 200
TEST - 1 (MOCK TEST - 1) Mark Scored : 103.33

21 He was a follower of Jainism in the early part of his career and converted to Saivism later. He is
hailed as ‘Vichitrachitta’ who constructed a temple for Brahma, Vishnu and Siva without the use of
bricks, timber, metal and mortar. Who is the ruler?

A. Kadambas of Banavasi
B. Adikavi Pampa
C. Rajaraja I
D. Mahendravarman I

Your Answer : D
Correct Answer : D

Answer Justification :

Learning: Mahendravarman I was converted to Saivism which led him to built a Siva temple at
Tiruvadi.
7
2 27
3
50 hails him as
He was a great builder of cave temples. The Mandagappattu inscription
1
4 Siva without the use of
Vichitrachitta who constructed a temple for Brahma, Vishnu9and
bricks, timber, metal and mortar. -
. com
ai l
gm
His rock-cut temples are found in a number of places like Tiruchirappalli.
a 7@
isr
mhis talents in painting. He is also regarded as an expert in
t
His title Chitrakarapuli reveals
i
music. lk u
- p
r a
is
The music inscription
M at Kudumianmalai is ascribed to him.

l k it
u
P Page 124: TN 11th Standard History Textbook
Q Source:

www.insightsias.com 25
© Insights Active Learning | All rights reserved - 78168. You may not reproduce, distribute or exploit the contents in any form without
written permission by copyright owner. Copyright infringers may face civil and criminal liability
Total Marks : 200
TEST - 1 (MOCK TEST - 1) Mark Scored : 103.33

22 Which of these justify the robust and benevolent system of administration that Cholas had?
1. Chola kings undertook royal tours to increase the efficiency of the administration.
2. All legal professions were exempt from tax liabilities.
3. All lands were carefully surveyed and classified for assessment of revenue.
4. During the hard times, there were remission of taxes.

Select the correct answer using the codes below.


A. 1 and 4 only
B. 1, 3 and 4 only
C. 2 and 3 only
D. 2 and 4 only

Your Answer : B
Correct Answer : B

Answer Justification :
7
2 27
Justification: Statement 1: There were regular drills with officials.
0 3
5
41
- 9 one place to another, various
Statement 2 and 3: There were tolls and customs on goods taken from
m like marriages and judicial fines.
kinds of professional taxes, dues levied on ceremonial occasions
i l .co
m a
Moreover, the land revenue department was well organized. It was called as
g
7@
puravuvarithinaikkalam.
r a
is called ur nattam. These and other lands such as the lands
The residential portion of the village was
m
k
belonging to temples were exempted
l it from tax.
- pu
sraearned the title – Sungam Tavirtta Cholan.
Statement 4: During the hard times, there were remission of taxes and Kulottunga I became famous
by abolishing tollsiand

k itM
The main
P ulitems of government expenditure were the king and his court, army and navy, roads,
irrigation tanks and canals.

Q Source: Page 146: TN 11th Standard History Textbook

www.insightsias.com 26
© Insights Active Learning | All rights reserved - 78168. You may not reproduce, distribute or exploit the contents in any form without
written permission by copyright owner. Copyright infringers may face civil and criminal liability
Total Marks : 200
TEST - 1 (MOCK TEST - 1) Mark Scored : 103.33

23 What is the significance of Sabarmati Ashram in the modern history of India?


1. It served as the last residence of Mahatma Gandhi.
2. It has been declared as a national monument by the Indian government.
3. Salt Satyagraha was started from this Ashram in 1930.

Select the correct answer using the codes below.


A. 1 only
B. 2 and 3 only
C. 3 only
D. 1 and 3 only

Your Answer : C
Correct Answer : B

Answer Justification :
7
27 was home to
Justification: The Sabarmati Ashram (also known as Harijan or Satyagraha Ashram)
2
03 of the Indian
Mohandas Gandhi from 1917 until 1930 and served as one of the main centres
5
freedom struggle. 1 4
-9
m Gandhiji left to Delhi from
Statement 1: This was Sevagram cottage (not Sabarmati Ashram).
o
c
l. there he did not return to Sewagram
Sevargam Cottage in 1946 and then went to Noakhali.iFrom
a
m making. He was martyred in Delhi in 1948.
while engaged in his task of communal unity and peace
7 @g
Statement 2 and 3: It is common knowledge.
isra
Learning: On his return from South k i tmAfrica, Gandhi’s first Ashram in India was established in the
Kochrab area of Ahmedabad p uinl 1915. The Ashram was then shifted in 1917 to a piece of open
a - of the river Sabarmati.
r
barren land on the banks
is
M
l k it
u
Reasons
Phusbandry,
for this shift included: he wanted to do some experiments in living eg farming, animal
cow breeding, Khadi and related constructive activities, for which he was in
search of this kind of barren land.

The Prime Minister has recently released the coin and postal stamp in the honour of Shrimad
Rajchandraji on his 150th birth anniversary at Abhay Ghat. Shrimad Rajchandraji was the
guru to Mahatma Gandhi.

Q Source: PM recently inaugurated the centenary celebrations of Sabarmati Ashram in Ahmedabad

www.insightsias.com 27
© Insights Active Learning | All rights reserved - 78168. You may not reproduce, distribute or exploit the contents in any form without
written permission by copyright owner. Copyright infringers may face civil and criminal liability
Total Marks : 200
TEST - 1 (MOCK TEST - 1) Mark Scored : 103.33

24 Which of these systems were popularly introduced by Alauddin Khalji?


1. Branding of horses
2. Maintaining a descriptive list of soldiers
3. Paying army in kind rather than cash
4. Market based pricing for all products

Select the correct answer using the codes below.


A. 1 and 2 only
B. 3 and 4 only
C. 2 and 4 only
D. 1 and 3 only

Your Answer : A
Correct Answer : A

Answer Justification :
7
2 27
5 03
Justification: Statement 1, 2 and 3: Alauddin Khalji maintained a large permanent standing army
and paid them in cash from the royal treasury.
9 41
-
m
l
He introduced the system of dagh (branding of horses)
i .co and prepared huliya (descriptive list of
soldiers).
m a
7 @g
isra a strict review of army from time to time was carried
In order to ensure maximum efficiency,
out.
k i tm
p ul
a -
i sr
Statement 4: The introduction of paying salaries in cash to the soldiers led to price regulations

k it Mas Market Reforms.


popularly called
l
Puwere under the control of a high officer called Shahna-i- Mandi. The supply of grain was
Markets
ensured by holding stocks in government store-houses.

Regulations were issued to fix the price of all commodities.

Learning: Every merchant was registered under the Market department. There were secret agents
called munhiyans who sent reports to the Sultan regarding the functioning of these markets.

Violation of regulations was severely punished. Harsh punishment was given if any shopkeeper
charged a higher price, or tried to cheat by using false weights and measures. Even during the
famine the same price was maintained

Q Source: Page 180: TN 11th Standard History Textbook

www.insightsias.com 28
© Insights Active Learning | All rights reserved - 78168. You may not reproduce, distribute or exploit the contents in any form without
written permission by copyright owner. Copyright infringers may face civil and criminal liability
Total Marks : 200
TEST - 1 (MOCK TEST - 1) Mark Scored : 103.33

25 Consider the following statements:


1. Under the Participatory Guarantee System for India (PGS-India), FAO bureaucracy certifies organic
products after due inspection.
2. National Programme for Organic Production (NPOP) intends to facilitate certification of organic
products in conformity with the importing countries organic standards.

Which of the above is/are correct?


A. 1 only
B. 2 only
C. Both 1 and 2
D. None

Your Answer : D
Correct Answer : B

Answer Justification : 7
2 27
Background: At present, the consumers do not have any method to check
5 03the authenticity of the
organic food products due to the absence of a regulatory framework. 1
4
-9
. com
a i l
FSSAI has come out with draft regulations for organic food products to ensure the
authenticity of organic foods.
g m
a 7@
FSSAI’s regulation mandates that
r
islabelling of organic foods should provide full and accurate
it m
information regarding thekorganic status of the organic foods.
l
- pu
ra
Organic foodisproducts must carry a certification mark or a quality assurance mark provided

k it M certification bodies.
by the notified
l
Pu
Justification: Statement 1: Organic foods will have to comply with the government’s provisions
under the National Programme for Organic Production (NPOP) or the Participatory Guarantee
System for India (PGS-India) run by the Agriculture Ministry or any other standards notified by
FSSAI.

The standards of Organic Crop Production have been described in NPOP document.

The production standards include crop production plan, conversion requirement, nutrient
management, disease management etc for agricultural crops.

Details will be covered in I-Learning (new initiative on website).

Statement 2: The PGS is an internationally applicable organic quality assurance system [like ISO
9000] implemented and controlled by the committed organic farmer-producers through active
participation, along with the consumers, in the process based on verifiable trust.
www.insightsias.com 29
© Insights Active Learning | All rights reserved - 78168. You may not reproduce, distribute or exploit the contents in any form without
written permission by copyright owner. Copyright infringers may face civil and criminal liability
Total Marks : 200
TEST - 1 (MOCK TEST - 1) Mark Scored : 103.33

It is not an “inspection raj” certification system but, rather, one that is based on personal integrity
and peer pressure. Integrity is honesty when no one is looking over your shoulder to see what you
are doing. So, 1 is wrong.

http://pgsindia-ncof.gov.in/pgs_india.aspx

Q Source: http://pib.nic.in/newsite/PrintRelease.aspx?relid=155000

http://www.thehindubusinessline.com/news/fssai-issues-draft-regulations-for-organic-food-products/
article9737535.ece

7
2 27
0 3
5
41
-9
. com
ai l
gm
7 @
isra
i t m
p ulk
a -
i sr
M
l k it
Pu

www.insightsias.com 30
© Insights Active Learning | All rights reserved - 78168. You may not reproduce, distribute or exploit the contents in any form without
written permission by copyright owner. Copyright infringers may face civil and criminal liability
Total Marks : 200
TEST - 1 (MOCK TEST - 1) Mark Scored : 103.33

26 Consider the following statements. Assertion (A): Kushanas banned the use of gold coins taken from
their empire on the silk route. Reason (R): Kushanas could not control the Silk route and were at a
loss from its trade.

In the context of the above, which of these is correct?


A. A is correct, and R is an appropriate explanation of A.
B. A is correct, but R is not an appropriate explanation of A.
C. A is correct, but R is incorrect.
D. Both A and R are incorrect.

Your Answer : D
Correct Answer : D

Answer Justification :

Justification: The best-known of the rulers who controlled the Silk Route were the Kushanas, who
ruled over central Asia and north-west India around 2000 years ago. 77 2
2
Their two major centres of power were Peshawar and Mathura. 5 03
1
94
- Asia down to the seaports at the
During their rule, a branch of the Silk Route extended from Central
m
il .co
mouth of the river Indus, from where silk was shipped westwards to the Roman Empire.
a
ofm
The Kushanas were amongst the earliest rulers g the subcontinent to issue gold. So, both A and R
@
are wrong.
s r a7
m i
Q Source: Page 102: NCERT Class
k t
i Our Pasts - I
VI
l
- pu
i sra
k itM
l
Pu

www.insightsias.com 31
© Insights Active Learning | All rights reserved - 78168. You may not reproduce, distribute or exploit the contents in any form without
written permission by copyright owner. Copyright infringers may face civil and criminal liability
Total Marks : 200
TEST - 1 (MOCK TEST - 1) Mark Scored : 103.33

27 Consider the following about ‘Know India Programme (KIP)’:


1. KIP is a programme run jointly by IISC, IITs and NITs to inculcate scientific temper and general
knowledge in youth.
2. KIP involves compulsory government financed field visits in public schools to places of national
importance.

Which of the above is/are correct?


A. 1 only
B. 2 only
C. Both 1 and 2
D. None

Your Answer :
Correct Answer : D

Answer Justification :
7
2 27
03 Affairs for the
Justification: Know India Programme is an initiative of the Ministry of External
5
Persons of Indian Origin (PIO) (including NRIs) between the age group
4 1 18 to 30 years to provide
of
them an exposure to the country of their origin (India). - 9
m
o their views, expectations and
i l
The KIP offers a platform for the young PIOs to visit India .cshare
experiences and forge closer bonds with the Indiam ofapresent times in a three-week orientation
programme.
7 @g
is ra
i t m launched a portal for the Know India Programme (KIP).
External Affairs Minister has recently

p ulk
Q Source: http://www.mea.gov.in/know-india-programme.htm
a -
is r
M
l k it
Pu

www.insightsias.com 32
© Insights Active Learning | All rights reserved - 78168. You may not reproduce, distribute or exploit the contents in any form without
written permission by copyright owner. Copyright infringers may face civil and criminal liability
Total Marks : 200
TEST - 1 (MOCK TEST - 1) Mark Scored : 103.33

28 Which of the two key principles/themes of democracy are inseparable or can’t exist without each
other?

A. Equity and Justice


B. Fraternity and Sovereignty
C. Accountability and transparency
D. Both (a) and (c)

Your Answer : D
Correct Answer : D

Answer Justification :

Justification: Option A: Justice implies fairness, it is a cardinal value of democracy. Equity implies
impartiality. Take Article 14 of constitution that ensures rule of law: it ensures equality between
citizens against law, that leads to justice for all.
7
2 27
Option B: There is no direct correlation between both. 3
1 50
4
Option C: You can take RTI as an example. If the government isn’t9transparent on information, it
-
om
can’t be held accountable. On the other hand, if there is no accountability, it will be most likely due
to opacity in functioning of government. Both go hand in.c hand.
ai l
m
Q Source: Improvisation: Page 40: NCERT Class
7 @g VI Social and Political Life - I
isra
k i tm
p ul
a -
isr
M
l k it
Pu

www.insightsias.com 33
© Insights Active Learning | All rights reserved - 78168. You may not reproduce, distribute or exploit the contents in any form without
written permission by copyright owner. Copyright infringers may face civil and criminal liability
Total Marks : 200
TEST - 1 (MOCK TEST - 1) Mark Scored : 103.33

29 Ramananda, Kabir and Nanak


1. Were not linked with any particular religious creed
2. Did not believe in rituals and ceremonies
3. Condemned monotheism and believed in multiple Gods
4. Denounced all forms of idolatry

Select the correct answer using the codes below.


A. 1, 2 and 3 only
B. 1, 2 and 4 only
C. 3 and 4 only
D. 1 and 3 only

Your Answer : B
Correct Answer : B

Answer Justification :
7
2 27
5 03
Justification: Statement 1: In the fourteenth and fifteenth centuries, Ramananda, Kabir and Nanak
remained great apostles of the Bhakti cult.
9 41
-
m
They drew inspiration from old masters but showed
i l .caonew path.
m a
g
@age-old
They helped the common people to shed a 7 superstitions and attain salvation through
Bhakti or pure devotion. is r
t m
u lki
-
Statement 2, 3 and 4: They pcondemned polytheism and believed in one god. They also denounced all
i srastrongly believed in Bhakti as the only means of salvation. They also
forms of idolatry. They

k it Mfundamental unity of all religions.


emphasised the
ul early reformers, they were not linked with any particular religious creed and did not
Pthe
Unlike
believe in rituals and ceremonies.

Q Source: Page 203: TN 11th Standard History Textbook

www.insightsias.com 34
© Insights Active Learning | All rights reserved - 78168. You may not reproduce, distribute or exploit the contents in any form without
written permission by copyright owner. Copyright infringers may face civil and criminal liability
Total Marks : 200
TEST - 1 (MOCK TEST - 1) Mark Scored : 103.33

30 Over the last decade, the country that is the World’s largest receiver of remittances is

A. USA
B. India
C. Russia
D. Canada

Your Answer : B
Correct Answer : B

Answer Justification :

Learning: According to UN International Fund for Agricultural Development (IFAD), Indians


working across the world had sent home USD 62.7 billion in 2016 making India the top remittance-
receiving country surpassing China.

2 77
2
03
The study is the first ever study conducted that analysed a 10-year trend in migration and
remittance flows over the period 2007-2016. 5
9 41
-
m
i l . co China to become the world’s
In the period between 2007 and 2016, India has surpassed
largest receiver of remittances.
m a
7 @g
isra
Asia remains the main remittance-receiving region with 55% of the global flows.

k i tm
p ul is expected to lift millions out of poverty and in attaining the
This increase in remittances
a -
i sr
Sustainable Development Goals (SDGs).

k itM
ul Remittance Receiving Countries: India (USD 62.7 billion), China (USD 61 billion), the
PTop
Philippines (USD 30 billion) and Pakistan (USD 20 billion).

Q Source: IFAD study as mentioned above

www.insightsias.com 35
© Insights Active Learning | All rights reserved - 78168. You may not reproduce, distribute or exploit the contents in any form without
written permission by copyright owner. Copyright infringers may face civil and criminal liability
Total Marks : 200
TEST - 1 (MOCK TEST - 1) Mark Scored : 103.33

31 Bhimbetka, in present-day Madhya Pradesh, is famous for

A. Hosting prehistoric cave paintings


B. Largest Buddha Bronze statues
C. Monolithic temples
D. None of the above

Your Answer : A
Correct Answer : A

Answer Justification :

Learning: Bhimbetka rock shelters are an archaeological site of the Paleolithic, exhibiting the
earliest traces of human life on the Indian Subcontinent, and thus the beginning of the Indian Stone
Age.

Some of the Stone Age rock paintings found among the Bhimbetka rock shelters 7 7 some 30,000
are
2 2
3 a World Heritage Site
years old. The caves also deliver early evidence of dance. They were declared
0
in 2003. 1 5
4
-9
om
Q Source: Page 15: NCERT Class VI Our Pasts - I
l . c
m ai
@ g
7
isra
it m
p ulk
a -
i sr
M
l k it
Pu

www.insightsias.com 36
© Insights Active Learning | All rights reserved - 78168. You may not reproduce, distribute or exploit the contents in any form without
written permission by copyright owner. Copyright infringers may face civil and criminal liability
Total Marks : 200
TEST - 1 (MOCK TEST - 1) Mark Scored : 103.33

32 Which of these types of trading/agreements in securities markets is NOT permitted in India?

A. Futures contract
B. Options contract
C. Forward contract
D. All are permitted

Your Answer : A
Correct Answer : D

Answer Justification :

Justification: Option A: Futures Contract has been permitted long back. It is a contract between
two parties where both parties agree to purchase or sell a particular commodity or any other
financial instrument at a predetermined price at a specified time in the future.

7
27 options
Option B: Commodity Exchanges have been requesting SEBI for a long time to permit
2
trading in commodities be allowed. It was permitted recently. 3 0
5
9 41
- the right to purchase without
m
Options Contract is a derivative product that offers an investor
any obligation to buy at the specified price/date. .co
a il
gm
@
7 as of now costs RM 100,000 (RM is a currency unit).
Suppose you want your dream car which
isra
it m
lkto buy the car now, but you will have after 3 months.
You don’t have the money
u
- p
sr a
Mi would be to buy an “option” to purchase this car in the future (after 3 months).
A smart move

l k it
Pu
You pay the car owner a contract premium (say RM 1,000) to exercise this option.

Remember, you have the option to buy or not buy the car.

See the diagram below now. Suppose after 3 months, car prices increase to RM 150,000. This
is a win case for you, since the options contract was to buy it at RM 100,000. You gain.

Suppose car price reduces. Now, you won’t buy the car, because the contract was not an
obligation (only a right) to buy the car. You would now pay the options premium (RM 1000 to
the owner), and may be buy the car from the market.

www.insightsias.com 37
© Insights Active Learning | All rights reserved - 78168. You may not reproduce, distribute or exploit the contents in any form without
written permission by copyright owner. Copyright infringers may face civil and criminal liability
Total Marks : 200
TEST - 1 (MOCK TEST - 1) Mark Scored : 103.33

77
22 regulated
Option C: You would heard of the Forward Markets Commission that earlier
3
forward markets. Now SEBI has subsumed the organization. 50 1
4
Q Source: -9
. com
http://www.livemint.com/Money/nfnDOuk11PlqWI43mgA1BL/Sebi-permits-options-trading-in-
commodity-futures.html ai l
gm
7 @
isra
i t m
p ulk
a -
i sr
M
l k it
Pu

www.insightsias.com 38
© Insights Active Learning | All rights reserved - 78168. You may not reproduce, distribute or exploit the contents in any form without
written permission by copyright owner. Copyright infringers may face civil and criminal liability
Total Marks : 200
TEST - 1 (MOCK TEST - 1) Mark Scored : 103.33

33 Which of these earliest cities was located to the east of Indus river?
1. Dholavira
2. Rakhi Garhi
3. Kalibangan
4. Sotkakoh

Select the correct answer using the codes below.


A. 1, 2 and 3 only
B. 2 and 4 only
C. 1 and 3 only
D. 1, 2, 3 and 4

Your Answer : A
Correct Answer : A

Answer Justification :
7
2 27
Justification:
0 3
5
41
-9
. com
ai l
gm
7 @
isra
i t m
p ulk
a -
i sr
M
l k it
Pu

Learning: These cities were found in the Punjab and Sind in Pakistan, and in Gujarat, Rajasthan,
Haryana and the Punjab in India.

Archaeologists have found a set of unique objects in almost all these cities: red pottery
painted with designs in black, stone weights, seals, special beads, copper tools, and long
www.insightsias.com stone blades. 39
© Insights Active Learning | All rights reserved - 78168. You may not reproduce, distribute or exploit the contents in any form without
written permission by copyright owner. Copyright infringers may face civil and criminal liability
Total Marks : 200
TEST - 1 (MOCK TEST - 1) Mark Scored : 103.33

Cities such as Kalibangan and Lothal had fire altars, where sacrifices may have been
performed.

In some cities, like Mohenjodaro, a very special tank, which archaeologists call the Great
Bath, was found.

Q Source: Page 33: NCERT Class VI Our Pasts - I

7
2 27
0 3
5
41
-9
. com
ai l
gm
7 @
isra
i t m
p ulk
a -
i sr
M
l k it
Pu

www.insightsias.com 40
© Insights Active Learning | All rights reserved - 78168. You may not reproduce, distribute or exploit the contents in any form without
written permission by copyright owner. Copyright infringers may face civil and criminal liability
Total Marks : 200
TEST - 1 (MOCK TEST - 1) Mark Scored : 103.33

34 The Ministry of Law and Justice has launched the ‘Tele-Law’ initiative in collaboration with the
Ministry of Electronics and Information Technology. Consider the following with reference to it.
1. It would subsume National Legal Service Authority (NALSA) and its State Level authorities to
create a Virtual Legal Service Authority (VLSA).
2. It will provide legal aid services to the marginalized communities and citizens living in rural areas.
3. It would help people to seek legal advice from lawyers with the help of video conferencing facility.

Select the correct answer using the codes below.


A. 1 and 2 only
B. 2 and 3 only
C. 3 only
D. 1, 2 and 3

Your Answer : B
Correct Answer : B

7
Answer Justification :
2 27
0 3
15(PLV), would be employed
Justification: Statement 2: Under the initiative, a Para Legal Volunteer
4
- 9 in blocks established with
in each Common Service Centres (CSC – which are IT service points
government support and regulation). m
i l .co
m a
@g of contact for the marginalized communities in
The volunteer would serve as the first point
7
a
the rural areas who will help themrunderstand the legal issues, explain the advice given by
is as per the advice of the lawyer.
lawyers and assist in further action
m
l k it
p u
a
Around 1000 women - PLVs will also be trained under this initiative to provide legal aid
r
is the CSCs.
services through
M
l k it
u
P 3: The National Legal Service Authority (NALSA) will also provide a panel of lawyers who
Statement
will provide advice from the state capitals to the applicants in the CSCs through video conferencing.

Learning: Apart from the lawyers, law school clinics, District Legal Service Authorities, voluntary
service providers and Non-Government Organisations working on legal aid and empowerment can
also be connected through the CSCs.

In the initial phase, the initiative will be tested as a pilot project across 500 Common Service
Centres (CSC) in Uttar Pradesh and Bihar.

The government will also put in place a robust monitoring and evaluation system to assess the
quality of the legal advice delivered to the people.

Q Source:
http://www.thehindu.com/news/national/india-launches-free-legal-aid-via-video-conferencing-to-villa
gers/article18970545.ece

www.insightsias.com 41
© Insights Active Learning | All rights reserved - 78168. You may not reproduce, distribute or exploit the contents in any form without
written permission by copyright owner. Copyright infringers may face civil and criminal liability
Total Marks : 200
TEST - 1 (MOCK TEST - 1) Mark Scored : 103.33

35 The Indian constitution prohibits discrimination between individuals on the basis of

A. Gender and caste only


B. Gender, caste, religion, race and place of birth
C. Gender, education, caste and place of birth
D. Caste, education, religion and race only

Your Answer : B
Correct Answer : B

Answer Justification :

Learning: These provisions come under the Right to Equality under fundamental rights.

The State shall not discriminate against any citizen on grounds only of religion, race, caste, sex,
place of birth or any of them.
2 77
No citizen shall, on grounds only of religion, race, caste, sex, place of birth 3 2
or any of them, be
subject to any disability, liability, restriction or condition with regard to50
9 41
-
m
access to shops, public restaurants, hotels and places
i l .coof public entertainment; or
m a
@g and places of public resort maintained wholly or
the use of wells, tanks, bathing ghats,7roads
i
partly out of State funds or dedicatedsra to the use of the general public.
k i tm
ul
Nothing in this article shallpprevent the State from making any special provision for women,
-
children, SCs or STs.ra
M is
k it
Q Source: Improvisation:
l Page 20: NCERT Class VI Social and Political Life - I
P u

www.insightsias.com 42
© Insights Active Learning | All rights reserved - 78168. You may not reproduce, distribute or exploit the contents in any form without
written permission by copyright owner. Copyright infringers may face civil and criminal liability
Total Marks : 200
TEST - 1 (MOCK TEST - 1) Mark Scored : 103.33

36 With reference to Ancient Southern India, Tolkappiyam refers to the five-fold division of lands.
Consider the following matches with their description.
1. Kurinji: Desert
2. Mullai: Pastoral
3. Neydal: Agricultural
4. Marudam: Hilly tracks
5. Palai: Coastal

Select the correct matches using the codes below.


A. 1, 3 and 4 only
B. 2 only
C. 1, 4 and 5 only
D. 1, 2, 3, 4 and 5

Your Answer :
Correct Answer : B
7
2 27
Answer Justification : 0 3
5
41
- 9 (pastoral), Marudam
Justification: This was the description: Kurinji (hilly tracks), Mullai
(agricultural), Neydal (coastal) and Palai (desert). m
i l .co
Learning: The people living in these five divisionsm a their respective chief occupations as well as
had
gods for worship.
7 @g
isra
k i tm – chief occupation, hunting and honey collection.
Kurinji – chief deity was Murugan
p ul
a -
i
Mullai – chief sr
deity Mayon (Vishnu) – chief occupation, cattle-rearing and dealing with dairy

k itM
products.
l
Pu
Marudam – chief deity Indira – chief occupation, agriculture.

Neydal – chief deity Varunan – chief occupation fishing and salt manufacturing.

Palai – chief deity Korravai – chief occupation robbery.

Q Source: Page 90: TN 11th Standard History Textbook

www.insightsias.com 43
© Insights Active Learning | All rights reserved - 78168. You may not reproduce, distribute or exploit the contents in any form without
written permission by copyright owner. Copyright infringers may face civil and criminal liability
Total Marks : 200
TEST - 1 (MOCK TEST - 1) Mark Scored : 103.33

37 “Kanishka must have conquered the greater part of the Gangetic plain”, the evidence of this
statement comes from
1. Coins of Kanishka are found in places like Mathura, Sravasti and Benares.
2. His coins exhibit only Buddhist images, and not Hindu gods, a cult that was popular then in the
Gangetic plain.

Which of the above is/are correct?


A. 1 only
B. 2 only
C. Both 1 and 2
D. None

Your Answer : C
Correct Answer : A

Answer Justification :
7
2 27
Justification: Statement 1: At the time of his accession his empire included 3Afghanistan, Gandhara,
Sind and Punjab. Subsequently he conquered Magadha and extended 1 50power as far as Pataliputra
his
4
and Bodh Gaya. This evidence comes from his coins that are found - 9in the gangetic plains.
c
Statement 2: Kanishka embraced Buddhism in the earlyl.part
om of his reign. However, his coins exhibit
the images of not only Buddha but also Greek andm ai gods. So, 2 is wrong.
Hindu

7 @g
sra Textbook
th
Q Source: Page 79: TN 11 Standard History
i
k i tm
p ul
a -
isr
M
l k it
Pu

www.insightsias.com 44
© Insights Active Learning | All rights reserved - 78168. You may not reproduce, distribute or exploit the contents in any form without
written permission by copyright owner. Copyright infringers may face civil and criminal liability
Total Marks : 200
TEST - 1 (MOCK TEST - 1) Mark Scored : 103.33

38 The Government has issued Public Procurement (Preference to Make in India), Order 2017 recently.
Consider the following with reference to its provisions:
1. The order applies to manufacturing sector only.
2. Ordinarily, all government procurement shall be based on 100% local content sourcing.
3. Import of non-essential commodities shall be severely regulated to promote domestic
manufacturing.

Select the correct answer using the codes below.


A. 1 only
B. 2 and 3 only
C. 3 only
D. None of the above

Your Answer : D
Correct Answer : D

7
Answer Justification :
2 27
0 3
Justification: This policy promotes manufacturing and production of 1 5 and services in India
goods
4
- 9are also covered, S1 is wrong.
with a view to enhancing income and employment. Since services

. com
In pursuance of this Order, purchase preference
l
aishall be given to local suppliers in all
m
@g in the manner specified in the order.
procurements undertaken by procuring entities
7
isra
As per the order the minimum
k i tmlocal content shall ordinarily be 50%. The Nodal Ministry may
ul percentage in respect of any particular item and may also
prescribe a higher or lower
p
a
prescribe the manner- of calculation of local content.
is r
M
l
Q Source: k it
http://dipp.nic.in/sites/default/files/publicProcurement_MakeinIndia_15June2017.pdf
Pu

www.insightsias.com 45
© Insights Active Learning | All rights reserved - 78168. You may not reproduce, distribute or exploit the contents in any form without
written permission by copyright owner. Copyright infringers may face civil and criminal liability
Total Marks : 200
TEST - 1 (MOCK TEST - 1) Mark Scored : 103.33

39 An important item of trade from Ancient India, it was highly valuable in Roman Empire so much that
it was known as ‘black gold’. What does the item refer to?

A. Crude oil
B. Coal
C. Dark Garnet stones
D. Pepper

Your Answer : D
Correct Answer : D

Answer Justification :

Learning: Frequently referred to as "black gold" in ancient India, on account of its demand and
trade value mainly due to its high trade value, the Zamorin ruler of Malabar coast had a flourishing
trade centre for export of pepper and other spices in Kerala.
7
2 27
The main mercantile traders were Arabs and people from Middle 5 0
East.
3
4 1
-9
c
The early Roman Empire got direct access to the Malabar
. om Coast in India and its range of
l
aiBC.
exotic spices after their conquest of Egypt in 30
gm
7 @
srahigh in the Middle Ages and the trade was completely
The prices of pepper were extremely
i
dominated by the Romans. tm
lk i
- pu
a
Q Source: Page 99: rNCERT
s Class VI Our Pasts - I
M i
l k it
Pu

www.insightsias.com 46
© Insights Active Learning | All rights reserved - 78168. You may not reproduce, distribute or exploit the contents in any form without
written permission by copyright owner. Copyright infringers may face civil and criminal liability
Total Marks : 200
TEST - 1 (MOCK TEST - 1) Mark Scored : 103.33

40 Consider the following:


1. Temple at Deogarh near Jhansi
2. Giant copper statue of Buddha originally found at Sultanganj
3. Delhi Iron pillar
4. Bagh caves paintings

The above belong to periods of which of these dynasties of India?


A. Peshwas
B. Guptas
C. Paramara
D. Western Ganga

Your Answer : B
Correct Answer : B

Answer Justification :
7
2 27
03 completely
Learning: The Delhi Iron pillar of the Gupta period is still free from rust though
5
exposed to sun and rain for so many centuries.
9 41
-
The paintings of the Gupta period are seen at Bagh caves near m Gwalior.
i l .co
a the life of the Buddha as depicted in the
Moreover, the mural paintings of Ajantha mostly illustrate
m
Jataka stories.
7 @g
There was little influence of Gandharais
ra
style on Gupta art. But the beautiful statue of standing
i m style. The Buddha statue unearthed at Saranath was
tGreek
k
Buddha at Mathura reveals a little
l
unique piece of Gupta art. pu
a -
is r
Q Source: Page 106-107: th
TN 11 Standard History Textbook
M
l k it
Pu

www.insightsias.com 47
© Insights Active Learning | All rights reserved - 78168. You may not reproduce, distribute or exploit the contents in any form without
written permission by copyright owner. Copyright infringers may face civil and criminal liability
Total Marks : 200
TEST - 1 (MOCK TEST - 1) Mark Scored : 103.33

41 Under the SMARTGRAM initiative of Rashtrapati (President) Bhavan, what is considered as a ‘smart
gram (village)’?

A. A village that is on the transition to become a municipal area


B. A village where a majority of the population is digitally literate
C. A village having required physical and social infrastructure governed with a layer of smart
information and communication systems
D. A village that is close to sub-urban areas of smart cities

Your Answer : C
Correct Answer : C

Answer Justification :

Learning: Under the project the selected villages will see new ventures in agriculture, skill
development, energy, education, heath, employment generation, and entrepreneurship.
277
2
03environment,
The model is based on the convergence of resources and effort by the central government, state
5
41
government, district administration, PRIs, private sector etc. to enhance the
connectivity and wellbeing of each village.
- 9
m
i l .co
Q Source: http://pib.nic.in/newsite/PrintRelease.aspx?relid=161641
m a
7 @g
isra
k i tm
p ul
a -
isr
M
l k it
Pu

www.insightsias.com 48
© Insights Active Learning | All rights reserved - 78168. You may not reproduce, distribute or exploit the contents in any form without
written permission by copyright owner. Copyright infringers may face civil and criminal liability
Total Marks : 200
TEST - 1 (MOCK TEST - 1) Mark Scored : 103.33

42 Consider the following statistics.


1. India ranks first among the world’s milk producing Nations.
2. Most of the milk produced in India comes from cooperatives.

Which of the above is/are correct?


A. 1 only
B. 2 only
C. Both 1 and 2
D. None

Your Answer : D
Correct Answer : A

Answer Justification :

Justification: Statement 1: India continued to be the largest milk producing nation (since 1998)
7
27the largest bovine
with milk production of nearly 155 million tonnes during 2015-16. India also has
2
population in the World. 03
4 15
- 9 in the domestic market, of
About 54% of the milk produced in the country is surplus for marketing
which only about 20% is procured/processed by the organised
o msector being equally, shared by
Cooperatives and Private dairy organizations. il. c
m a
g
7@
Statement 2: The dairy cooperatives collectively procure around 16 million tonnes of milk.
r a
sdouble
Learning: Government is mandated to
m i milk producers’ income at farm level by providing
t
lki access to the organised milk processing sector.
rural milk producers with greater
u
-p
Department of AnimalaHusbandry, Dairying & Fisheries has formulated a Draft National Action Plan
i sr recently.
for Dairy Development

k itM
l PIB SS (Release ID :167289)
Pu
Q Source:

www.insightsias.com 49
© Insights Active Learning | All rights reserved - 78168. You may not reproduce, distribute or exploit the contents in any form without
written permission by copyright owner. Copyright infringers may face civil and criminal liability
Total Marks : 200
TEST - 1 (MOCK TEST - 1) Mark Scored : 103.33

43 With reference to the Indus Valley civilization, consider the following statements:
1. Indus valley civilization was the biggest exporter of semi-precious stones and a major importer of
agricultural products.
2. Fishing was a regular occupation while hunting and bull fighting were banned.
3. Linga worship was prevalent and even natural figures like trees were worshipped.
4. Lothal was an important point of trade between the Harappan civilization and the remaining part of
India as well as Mesopotamia.

Select the correct answer using the codes below.


A. 1 and 2 only
B. 3 and 4 only
C. 1, 2 and 3 only
D. 2, 3 and 4 only

Your Answer : B
Correct Answer : B
7
2 27
Answer Justification : 0 3
5
41
- 9 stones were imported. Main
Justification: Statement 1: Gold, copper, tin and several semi-precious
exports were several agricultural products such as wheat, o m peas, oil seeds etc.
barely,

a il.c
Statement 2: Hunting and bull fighting were common
g m pastimes, not banned. Marbles, balls and dice
were also used for games.
a 7@
r
is the Mother Goddess represented in terracotta figurines. In
Statement 3: The chief female deitym was
latter times, Linga worship wasl k it
prevalent. Trees and animals were also worshipped by the
u
p in ghosts and evil forces and used amulets as protection against
Harappans. They also believed
-
them.
i sra
M
Statement 4:itLothal is an important Harappan centre, you should read more about it.
l k
Pu
Q Source: Page 21-23: TN 11th Standard History Textbook

www.insightsias.com 50
© Insights Active Learning | All rights reserved - 78168. You may not reproduce, distribute or exploit the contents in any form without
written permission by copyright owner. Copyright infringers may face civil and criminal liability
Total Marks : 200
TEST - 1 (MOCK TEST - 1) Mark Scored : 103.33

44 Many Chinese Buddhist pilgrims visited Indian subcontinent to visit places associated with the life of
the Buddha. Who among the following is NOT one of them?

A. Fa Xian
B. Xuan Zang
C. I-Qing
D. Alan Chen

Your Answer : D
Correct Answer : D

Answer Justification :

Learning: Three such Chinese Buddhist pilgrims are best-known.

Fa Xian, who came to the subcontinent about 1600 years ago, Xuan Zang (who came around 1400
7
years ago) and I-Qing, who came about 50 years after Xuan Zang are the famous7one.
3 22
They have documented previous socio-economic, political and religious 5 0 of Indian life then.
details
1
- 94
We will cover all travellers in detail in coming tests.
. com
Q Source: Page 105: NCERT Class VI Our Pasts - I ai l
gm
a 7@
is r
it m
l k
- pu
i sra
k itM
l
Pu

www.insightsias.com 51
© Insights Active Learning | All rights reserved - 78168. You may not reproduce, distribute or exploit the contents in any form without
written permission by copyright owner. Copyright infringers may face civil and criminal liability
Total Marks : 200
TEST - 1 (MOCK TEST - 1) Mark Scored : 103.33

45 Consider the following statements.


1. Anyone who is a resident of the concerned village is a member of the Gram Sabha.
2. All plans for the work of the Gram Panchayat are placed before the people in Gram Sabha.

Which of the above is/are correct?


A. 1 only
B. 2 only
C. Both 1 and 2
D. None

Your Answer : A
Correct Answer : B

Answer Justification :

Justification: Statement 1: Only one who is 18 years old or more and who has the right to vote is a
7
member of the Gram Sabha. 27
0 32
The Gram Sabha is a meeting of all adults who live in the area covered 5 a Panchayat. This could
1by
4
be only one village or a few villages. -9
. com
Statement 2: The Gram Sabha prevents the Panchayat from
a i l doing wrong things like misusing
money or favouring certain people.
gm
a 7on@the elected representatives and in making them
r
It plays an important role in keeping an eye
isthem.
responsible to the persons who elected
it m
l k
u 43-45: NCERT Class VI Social and Political Life - I
Q Source: Improvisation: p Page
-
i sra
k itM
l
Pu

www.insightsias.com 52
© Insights Active Learning | All rights reserved - 78168. You may not reproduce, distribute or exploit the contents in any form without
written permission by copyright owner. Copyright infringers may face civil and criminal liability
Total Marks : 200
TEST - 1 (MOCK TEST - 1) Mark Scored : 103.33

46 With reference to the Vedic civilization, Rajasuya, Asvamedha and Vajpeya were

A. Rituals and sacrifices performed by the King to strengthen his position


B. Mercenary army contingents that were owned by the local landlords
C. Land tax imposed on areas inside the jurisdiction of the Empire
D. Body of officials who maintained a record of the extent of empire, its defeats and successions

Your Answer : A
Correct Answer : A

Answer Justification :

Context: Larger kingdoms were formed during the later Vedic period as many jana or tribes were
amalgamated to form janapadas or rashtras in the later Vedic period.

Hence the royal power had increased along with the increase in the size of kingdom.
2 77
2
03Vajpeya (chariot race).
Learning: So, the king performed various rituals and sacrifices to strengthen his position. They
include Rajasuya (consecration ceremony), Asvamedha (horse sacrifice)5and
9 41
-
The kings also assumed titles like Rajavisvajanan, Ahilabhuvanapathi, (lord of all earth), Ekrat and
o m
il.c
Samrat (sole ruler) to showcase his power.

m a
th
g
Q Source: Page 32: TN 11 Standard History Textbook
a 7@
is r
it m
l k
- pu
i sra
k itM
l
Pu

www.insightsias.com 53
© Insights Active Learning | All rights reserved - 78168. You may not reproduce, distribute or exploit the contents in any form without
written permission by copyright owner. Copyright infringers may face civil and criminal liability
Total Marks : 200
TEST - 1 (MOCK TEST - 1) Mark Scored : 103.33

47 Jainism advocates three principles known as Triratnas (three gems) which are right faith, right
knowledge and right conduct. On the other hand, Buddhism preaches four noble truths and the
eightfold path. What is/are common between the teachings of both religions?
1. Acceptance of the theory that God exists and it has multiple aspects
2. Understanding that no object possesses a soul and is inanimate in the larger flow of life
3. To practice ahimsa and abstain from acquiring precious metals like Gold and Silver
4. Penance and extreme austerity to punish the body

Select the correct answer using the codes below.


A. 1 and 2 only
B. 3 only
C. 2 and 3 only
D. 1 and 4 only

Your Answer : B
Correct Answer : B
7
2 27
Answer Justification :
0 3
5
9
Justification: We will explain it in the light of teachings of Mahavira. 41
-
m
Justification: Statement 1 and 2: Right Knowledge (aslper
i .coMahavira) is the acceptance of the
theory that there is no God and that the world has been
m a existing without a creator and that all
objects possess a soul.
7 @g
Buddhism sheds any discussion on God i ra soul as futile. Buddha also taught that the soul does not
sand
k
exist, so there is no meeting ground i tmin this case. So, 1 and 2 are wrong.
p ul
Statement 3: Right conducta - (Jainism) refers to the observance of the five great vows such as not to
r
is not to steal, not to acquire property and not to lead an immoral life.
injure life, not to lie,
M
l k it forbids its monks from acquiring property (that may bring a sense of ego and pride
Buddhism u also
to its P
monks and nuns) and injuring life. So, 3 is a meeting ground.

Statement 4: Only Jainism advocates extreme penance, Buddhism preaches moderation. So, 4 is
wrong.

Q Source: Page 39: TN 11th Standard History Textbook

www.insightsias.com 54
© Insights Active Learning | All rights reserved - 78168. You may not reproduce, distribute or exploit the contents in any form without
written permission by copyright owner. Copyright infringers may face civil and criminal liability
Total Marks : 200
TEST - 1 (MOCK TEST - 1) Mark Scored : 103.33

48 Which of these is NOT one of the major styles or schools of ancient Indian art?

A. Amravati
B. Mathura
C. Gandhara
D. Sarnath

Your Answer : D
Correct Answer : D

Answer Justification :

Learning: The Amravati school of art had great influence on art in Sri Lanka and South-East Asia
as products from here were carried to those countries. It also had influence over South Indian
sculpture.

77 mainly
Mathura: During the first century AD, Gandhara and Mathura School of Art flourished
2
during reign of Kushana emperor Kanishka. 3 2
5 0
41
Mathura School had developed indigenously. -9
. com
ai l
gmof this art were Sarnath and Kosambi.
Apart from Mathura, other important centres
a 7@
isr
tm was the spotted red sandstone. This art reached its peak
The material used in this school
ki6th
during the Gupta periodlin or 7th century.
p u
a -
is r
Gandhara will be covered later.
k itM
l
Pu Improvisation: Page 66: NCERT Class VI Our Pasts - I
Q Source:

www.insightsias.com 55
© Insights Active Learning | All rights reserved - 78168. You may not reproduce, distribute or exploit the contents in any form without
written permission by copyright owner. Copyright infringers may face civil and criminal liability
Total Marks : 200
TEST - 1 (MOCK TEST - 1) Mark Scored : 103.33

49 With reference to ANUGA 2017, consider the following:


1. It is an inter-governmental platform organized by UNSC for promoting defence technology
cooperation.
2. India will be a co-Partner Country in 2017 edition of the event.
3. Agreements signed under the event are mandated to take the place of national laws and impose
regulatory burdens on party nations.

Select the correct answer using the codes below.


A. 1 only
B. 2 and 3 only
C. 2 only
D. 3 only

Your Answer :
Correct Answer : C
7
Answer Justification :
2 27
0 3
4 15
Justification: Statement 1: ANUGA is the world’s biggest and most important trade fair for Food
and beverage trade held in Germany. - 9
. com
ai l
It takes place biennially (every 2 years). ANUGA 2017 is the 34th edition.
gm
@
s r a7
i
ANUGA offers an extensive supporting programme with Lectures, Special exhibition and
attractive industry events itm
p ulk
a -
i
More than fiftysrper cent of the industry visitors are foreigners.

k itM
l
Pu 2: India is to become a co-Partner Country in ANUGA 2017. It gives India an opportunity
Statement
to showcase its strengths and opportunities to an international audience in the food sector and
invite investments.

Statement 3: There is no such provision.

Q Source: http://pib.nic.in/newsite/PrintRelease.aspx?relid=165597

www.insightsias.com 56
© Insights Active Learning | All rights reserved - 78168. You may not reproduce, distribute or exploit the contents in any form without
written permission by copyright owner. Copyright infringers may face civil and criminal liability
Total Marks : 200
TEST - 1 (MOCK TEST - 1) Mark Scored : 103.33

50 Indian Railways Organization for Alternate Fuel (IROAF) has been recently awarded the coveted
National level “Golden Peacock Award for the Year 2017 for Eco-Innovation”. The award was given
for
1. Substitution of Diesel in passenger trains by environment friendly Compressed Natural Gas (CNG).
2. Installation of open source licensing system for electricity generation and procurement

Which of the above is/are correct?


A. 1 only
B. 2 only
C. Both 1 and 2
D. None

Your Answer : D
Correct Answer : A

Answer Justification :
2 77
Justification: Indian Railways has achieved a major landmark in the field of 2
3 Eco friendly fuel
5 0
41
technologies by this innovation.
9
-done for the first time in the world.
Use of CNG in Train Sets for passenger transportation has been m
i l .co
a
mCNG)
This innovation (20% substitution of dieselgby will reduce emissions considerably by
@
bringing down NOx, CO2 and Particulate
s r a7 Matter besides achieving economy in fuel cost by
eight per cent.
m i
t
u lki
p thirteen hundred crores annually if implemented over entire fleet of
It could save up to-over
i sra of Indian Railways.
Diesel Locomotives

k itM
l
Pu technology of 40% substitution is at present being developed by IROAF. This will
An improved
enhance the potential of saving of fuel cost of IR to about Rs. thirty four hundred crore per annum.

Q Source: PIB Releases: 11th July

www.insightsias.com 57
© Insights Active Learning | All rights reserved - 78168. You may not reproduce, distribute or exploit the contents in any form without
written permission by copyright owner. Copyright infringers may face civil and criminal liability
Total Marks : 200
TEST - 1 (MOCK TEST - 1) Mark Scored : 103.33

51 The Mauryan state had a well-organized civil service, this is evident from
1. Civil servants called Amatyas who looked after the day-to-day administration were selected based
on merit.
2. Adyakshas officers helped control the retail and wholesale prices of goods and ensured their steady
supply in the state.
3. All officials were interviewed and appointed directly by the emperor.

Select the correct answer using the codes below.


A. 1 and 2 only
B. 2 and 3 only
C. 3 only
D. 2 only

Your Answer : A
Correct Answer : A
7
Answer Justification :
2 27
0 3
15king in administrative
Justification: A council of ministers called Mantriparishad assisted the
4
matters. -9
om
il.c
It consisted of Purohita, Mahamantri, Senapati and Yuvaraja.
a
m
g of independent India. The method of selection
Statement 1: Amatyas were similar to the IAS@ officers
7
sra
of Amatyas was elaborately given by Kautilya.
i
k i
Statement 2: Department of Commerce tm and Industry had controlled the retail and wholesale prices
l steady supply through its officers called Adyakshas. It also
utheir
p
of goods and tried to ensure
-
controlled weights and
r a measures, levied custom duties and regulated foreign trade.
M is
Statement 3:itSelection process was different for different officials.
u lk
P Asoka also appointed Dhamma Mahamatras to supervise the spread of Dhamma.
Learning:

Even the military service was well organized. The Mauryan army was well organized and it was
under the control of Senapati. The salaries were paid in cash. Kautilya refers to the salaries of
different ranks of military officers.

Q Source: Page 66-67: TN 11th Standard History Textbook

www.insightsias.com 58
© Insights Active Learning | All rights reserved - 78168. You may not reproduce, distribute or exploit the contents in any form without
written permission by copyright owner. Copyright infringers may face civil and criminal liability
Total Marks : 200
TEST - 1 (MOCK TEST - 1) Mark Scored : 103.33

52 Ptolemy, a Greek writer, mentions many important ports in the Deccan. Among them, the greatest
port of the Satavahanas in Western Deccan was

A. Kalyani
B. Ganjam
C. Satakarni
D. Pulamayi

Your Answer : B
Correct Answer : A

Answer Justification :

Learning: Vashishtaputra Pulamayi extended the Satavahana power up to the mouth of the Krishna
river.

7 and
He issued coins on which the image of ships was inscribed. They reveal the naval7power
2 2
maritime trade of the Satavahanas. 3
1 50
Their greatest port was Kalyani on the west Deccan. Gandakasela 9 4 Ganjam on the east coast
and
-
om
were the other important seaports.
l . c
ai
The last great ruler of Satavahanas was Yajna Sri Satakarni.
m
g
@Textbook
7
sra
th
Q Source: Page 76: TN 11 Standard History
i
k i tm
p ul
a -
is r
M
l k it
Pu

www.insightsias.com 59
© Insights Active Learning | All rights reserved - 78168. You may not reproduce, distribute or exploit the contents in any form without
written permission by copyright owner. Copyright infringers may face civil and criminal liability
Total Marks : 200
TEST - 1 (MOCK TEST - 1) Mark Scored : 103.33

53 The National Mission on Cultural Mapping of India intends to


1. Create repository of information about cultural assets
2. Launch massive cultural awareness campaigns
3. Open a direct channel of communication of artists with the Government
4. Hold talent hunt competitions from Block level to National level

Select the correct answer using the codes below.


A. 1 and 4 only
B. 2 and 3 only
C. 1, 2 and 3 only
D. 1, 2, 3 and 4

Your Answer : A
Correct Answer : D

Answer Justification : 7
2 27
Learning: The mission aims at converting the vast and widespread cultural
5 03canvas of India into an
objective cultural map to preserve our cultural heritage.
9 41
-
m
The Mission also seeks to open a direct channel of
i l co
.communication of artists with the
Government and peer to peer communication a
mamong artists for talent honing and handholding
g
7@
of each other.
r a
m is
k
With a vision that no cultural
l it talent or heritage should remain unattended (so that it could be
puof Culture has planned to hold talent hunt competitions with various
nurtured), the Ministry
-
sra
rounds from Block level to National level.
i
k itM
Thel Artist/Organisations also may register online at www.culturalmapping.nic.in.
Pu
The Cultural Mapping of India portal will serve as a repository/consolidated database of
information about cultural assets and resources i.e. a cultural inventory at one place for
essential planning, sustainable economic development and for preserving the scattered and
near extinct art forms.

Q Source: http://pib.nic.in/newsite/PrintRelease.aspx?relid=165655

www.insightsias.com 60
© Insights Active Learning | All rights reserved - 78168. You may not reproduce, distribute or exploit the contents in any form without
written permission by copyright owner. Copyright infringers may face civil and criminal liability
Total Marks : 200
TEST - 1 (MOCK TEST - 1) Mark Scored : 103.33

54 This council held near Srinagar prepared an authoritative commentary on the Buddhist Tripitakas
and gave the Mahayana doctrine a final shape. The council was convened by

A. Kanishka
B. Samprati
C. Bindusara Amitraghata
D. Devavarman

Your Answer : A
Correct Answer : A

Answer Justification :

Learning: He convened this Fourth Buddhist Council that discussed matters relating to Buddhist
theology and doctrine.

277
It was held at the Kundalavana monastery near Srinagar in Kashmir under the presidentship of
Vasumitra. About 500 monks attended the Council. 3 2
1 50
Learning: Asvagosha was a great philosopher, poet and dramatist9and 4 was particularly close to
Kanishka. He became his religious advisor in his later years m -
and composed Buddhacharita.
o
Nagarjuna from south India also adorned the court of a l.c
iKanishka. The famous physician of ancient
m
India Charaka was also patronized by him.
7 @g
th
i
Q Source: Page: TN 11 Standard HistorysraTextbook
k i tm
p ul
a -
isr
M
l k it
Pu

www.insightsias.com 61
© Insights Active Learning | All rights reserved - 78168. You may not reproduce, distribute or exploit the contents in any form without
written permission by copyright owner. Copyright infringers may face civil and criminal liability
Total Marks : 200
TEST - 1 (MOCK TEST - 1) Mark Scored : 103.33

55 Consider the following about The Energy Resources Institute (TERI).


1. It is a non-profit policy research organisation.
2. It is an attached agency of the International Centre for Climate Governance (ICCG).

Which of the above is/are correct?


A. 1 only
B. 2 only
C. Both 1 and 2
D. None

Your Answer : D
Correct Answer : A

Answer Justification :

Justification: Statement 1: TERI conducts research work in the fields of energy, environment and
7
sustainable development. 27
0 32
Statement 2: The Energy Resources Institute (TERI) was ranked second
4 15among world’s best climate
-9
think tank by the International Centre for Climate Governance (ICCG).

. com
ICCG is an internationally renowned centre founded in l2009.
m ai
@g climate policy and related governance issues.
The main focus of ICCG activities is the design of
7
isra
Q Source: www.insightsonindia.com/2017/07/06/insights-daily-current-affairs-06-july-2017/
k i tm
p ul
a -
is r
M
l k it
Pu

www.insightsias.com 62
© Insights Active Learning | All rights reserved - 78168. You may not reproduce, distribute or exploit the contents in any form without
written permission by copyright owner. Copyright infringers may face civil and criminal liability
Total Marks : 200
TEST - 1 (MOCK TEST - 1) Mark Scored : 103.33

56 Which of these ancient scripts generally writes in left to right direction?

A. Brahmi
B. Kharosthi
C. Harappan
D. All of the above

Your Answer : A
Correct Answer : A

Answer Justification :

Learning: Brahmi: is one of the oldest writing systems used in South and Central Asia from the 1st
millennium BCE.

Most modern Indian scripts have developed from the Brahmi script over hundreds of years.
2 77
2
03
Harappan: Iravatham Mahadevan established that the Harappan script is from right to left.
5
41
Kharosthi: is an ancient script used in ancient Gandhara (primarily9modern-day Afghanistan and
Pakistan) to write the Gandhari Prakrit and Sanskrit. -
. com
Q Source: Improvisation: Page 81: NCERT Class VI a l
i Pasts - I
Our
m
7 @g
isra
k i tm
p ul
a -
is r
M
l k it
Pu

www.insightsias.com 63
© Insights Active Learning | All rights reserved - 78168. You may not reproduce, distribute or exploit the contents in any form without
written permission by copyright owner. Copyright infringers may face civil and criminal liability
Total Marks : 200
TEST - 1 (MOCK TEST - 1) Mark Scored : 103.33

57 Consider the following statements.


1. Innovate in India (i3) program offers young entrepreneurs an avenue to engage with the best
practices in the domestic financial and banking industry.
2. National Biopharma Mission will concentrate, inter alia, on development of medical devices and
biotherapeutics to upgrade India’s product development capabilities to globally competitive levels.

Which of the above is/are correct?


A. 1 only
B. 2 only
C. Both 1 and 2
D. None

Your Answer : D
Correct Answer : B

Answer Justification : 7
2 27
Justification: Statement 1: i3 is a flagship program of the Government of0India 3 in collaboration
5
with World Bank.
9 41
-
o m
It is committed to make India a hub for design and
i l .cdevelopment of novel, affordable and
m
effective biopharmaceutical products and solutionsa by engaging with entrepreneurs.
g
a 7@
r
is biopharmaceutical market share from current 2.8% to
It will strive to increase India’s global
m
itbusiness opportunity of $16 billion.
5% and generate additional
l k
- pu
Statement 2 (S2): iItsr
a
will bring together expertise from national and international corridors to

k i t Mdirection and guidance to move promising solutions through the product


provide strategic
l value chain.
development
Pu
The mission will be implemented by Biotechnology Industry Research Assistance Council (BIRAC), a
PSU of Department of Biotechnology.

Besides, it will also work on establishment of shared infrastructure and facilities and at the same
time creating and enhancing technology transfer capabilities in public and private sector.

Q Source: http://pib.nic.in/newsite/PrintRelease.aspx?relid=166951

www.insightsias.com 64
© Insights Active Learning | All rights reserved - 78168. You may not reproduce, distribute or exploit the contents in any form without
written permission by copyright owner. Copyright infringers may face civil and criminal liability
Total Marks : 200
TEST - 1 (MOCK TEST - 1) Mark Scored : 103.33

58 In a democratic state, government derives its ultimate sovereignty from

A. Citizens of the state


B. Territory of the State
C. Laws governing the state
D. Parliament

Your Answer : A
Correct Answer : A

Answer Justification :

Justification: The chain is straightforward.

People elect representatives, representatives form Parliament, Parliament chooses the government,
government makes laws, and those laws regulate the territory of the state.
7
2 27
So, the root of sovereignty can be easily traced to people. 3
1 50
4 Life - I
-9
Q Source: Improvisation: Page 31: NCERT Class VI Social and Political

. com
ai l
m
7 @g
isra
k i tm
p ul
a -
isr
M
l k it
Pu

www.insightsias.com 65
© Insights Active Learning | All rights reserved - 78168. You may not reproduce, distribute or exploit the contents in any form without
written permission by copyright owner. Copyright infringers may face civil and criminal liability
Total Marks : 200
TEST - 1 (MOCK TEST - 1) Mark Scored : 103.33

59 Chandragupta II, a ruler of the Gupta Empire, assumed the famous title of ‘Sakari’. What did the title
imply?

A. Destroyer of Sakas
B. Lord of the world
C. Well Wisher of masses
D. Devotee of Shankara

Your Answer : B
Correct Answer : A

Answer Justification :

Learning: The greatest of the military achievements of Chandragupta II was his war against the
Saka satraps of western India.

2 77
32 and killed.
Rudrasimha III, the last ruler of the Saka satrap was defeated, dethroned
0
5
9 41
- were annexed into the Gupta
His territories in western Malwa and the Kathiawar Peninsula
m
Empire. co
il.
m a
g
After this victory he performed the horse
a 7@sacrifice and assumed the title Sakari, meaning,
is
‘destroyer of Sakas’. He also called r himself Vikramaditya.

k i tm
l
uStandard
Q Source: Page 101: TN 11
- p th
History Textbook

i sra
k itM
l
Pu

www.insightsias.com 66
© Insights Active Learning | All rights reserved - 78168. You may not reproduce, distribute or exploit the contents in any form without
written permission by copyright owner. Copyright infringers may face civil and criminal liability
Total Marks : 200
TEST - 1 (MOCK TEST - 1) Mark Scored : 103.33

60 Consider the following statements about the recently established Global Foreign Exchange
Committee (GFXC).
1. It was established as a research arm of International Monetary Fund (IMF).
2. GFXC will enforce a FX Global Code that will legally regulate the manipulation of floating foreign
exchange by Central Banks.
3. It will promote collaboration and communication among the local foreign exchange committees
(FXCs) of member nations.
4. It will replace the institution of Bank of International Settlements (BIS).
5. Indian government has boycotted GFXC due to its non-democratic governance structure.

Select the correct answer using the codes below.


A. 2, 3 and 4 only
B. 1, 2 and 5 only
C. 3 only
D. 1, 2, 3 and 4 only

7
Your Answer : C
2 27
Correct Answer : C 0 3
5
41
Answer Justification : -9
om
il.c the long explanation.
Justification: This is a very important development, hence
a
m
Statement 1: GFXC was established in 2017 as
7 @agforum bringing together central banks and private
sector participants with the aim to promote
is ra a robust, liquid, open, and appropriately transparent FX
market.
k i tm
Statement 2: FX Global Codep uisl a set of global principles of good practice in the foreign exchange
market, developed tora
-
provide a common set of guidelines to promote the integrity and effective
s
functioning of the iwholesale foreign exchange market.
t M
u lki
PThe Global Code does not impose legal or regulatory obligations on Market Participants, nor
does it substitute for regulation.

It is rather intended to serve as a supplement to any and all local laws, rules and regulations
by identifying global good practices and processes.

Statement 3: The objectives of the GFXC are:

to promote collaboration and communication among the local foreign exchange committees
(FXCs) and non-GFXC jurisdictions with significant FX markets;

to exchange views on trends and developments in global FX markets, including on the


structure and functioning of those markets, drawing on information gathered at the various
FXCs; and

www.insightsias.com 67
© Insights Active Learning | All rights reserved - 78168. You may not reproduce, distribute or exploit the contents in any form without
written permission by copyright owner. Copyright infringers may face civil and criminal liability
Total Marks : 200
TEST - 1 (MOCK TEST - 1) Mark Scored : 103.33

to promote, maintain and update on a regular basis the FX Global Code (the Code) and to
consider good practices regarding effective mechanisms to support adherence.

Statement 4: The GFXC committee has been set up under guidance of the Bank for International
Settlements (BIS), an international financial organisation.

Statement 5: India will soon join Global Foreign Exchange Committee (GFXC).

Q Source:
http://www.thehindubusinessline.com/markets/forex/india-to-join-new-global-foreign-exchange-com
mittee/article9756342.ece

7
2 27
0 3
5
41
-9
. com
ai l
gm
7 @
isra
i t m
p ulk
a -
i sr
M
l k it
Pu

www.insightsias.com 68
© Insights Active Learning | All rights reserved - 78168. You may not reproduce, distribute or exploit the contents in any form without
written permission by copyright owner. Copyright infringers may face civil and criminal liability
Total Marks : 200
TEST - 1 (MOCK TEST - 1) Mark Scored : 103.33

61 Why the sale of Oxytocin, despite being a natural hormone, is heavily regulated?
1. It was used as a depressant by a large segment of population.
2. Its indiscriminate use by dairy farmers was causing irreversible hormone damage in milch animals.
3. Consumption of food laced with Oxytocin can cause cardiac problems in humans.
4. It is abused to speed up child birth in women.

Select the correct answer using the codes below.


A. 1 and 3 only
B. 2, 3 and 4 only
C. 2 and 4 only
D. 1, 2, 3 and 4

Your Answer : B
Correct Answer : B

Answer Justification :
7
2 27
Justification: Statement 1: Oxytocin has also been dubbed the love or the 3
5 0 bliss hormone due to its
effects on behaviour. It isn’t a depressant. So, 1 is wrong.
9 41
-
o m
Statement 2: The drug Oxytocin is banned in India under Prevention of Cruelty to Animals Act and
Food and Drug Adulteration Prevention Act, 1960. il. c
m a
g
a
In mammals, Oxytocin stimulates ejection 7@ of milk from the mammary glands.
is r
t m
But, its use can cause u lki
reproductive damage in animals and even their death.
- p
r a
M is of the drug in humans can lead to haemorrhage, palpitation, low blood
Statement 3: Overdose
k
pressure landitin some cases even death.
Pu
Statement 4: The release of oxytocin by the pituitary gland acts to regulate two female reproductive
functions: Childbirth and Breast-feeding. So, it is used to speed up child birth which is wrong and
should be shunned.

So, after being banned in retail markets, the Indian Drug Controller will soon restrict
manufacturing of controversial hormone drug Oxytocin to public sector undertakings (PSUs).

Q Source:
http://www.thehindu.com/news/national/only-psus-may-be-let-to-make-oxytocin/article18868286.ece

www.insightsias.com 69
© Insights Active Learning | All rights reserved - 78168. You may not reproduce, distribute or exploit the contents in any form without
written permission by copyright owner. Copyright infringers may face civil and criminal liability
Total Marks : 200
TEST - 1 (MOCK TEST - 1) Mark Scored : 103.33

62 Consider the following statements.


1. Manimegalai is a Tamil epic that criticizes Buddhism while investigating the strengths of
contemporary Hindu traditions.
2. Silappatikaram, produced at Sangam, is a poetic rendition with details of Tamil culture.

Which of the above is/are correct?


A. 1 only
B. 2 only
C. Both 1 and 2
D. None

Your Answer : B
Correct Answer : B

Answer Justification :
7
27Pattuppattu,
Justification: The corpus of Sangam literature includes Tolkappiyam, Ettutogai,
2
Pathinenkilkanakku, and the two epic – Silappathigaram and Manimegalai. 3
5 0
41
-9
Statement 1: The aim of the author of Manimegalai, Seethalai Saathanar, was to compare Buddhism
favourably with the other prevailing religions in South India m
in order to propagate Buddhism.
i l .co
m a
@ g
He criticizes Hinduism, and exposes the weaknesses of the other contemporary Indian
7
a Teaching, the Dharma, as the most perfect religion. So,
religions, while praising the Buddha's
1 is wrong. isrm
l k it
p u
Statement 2: Regarded as
a - one of the great works of Tamil literature, the Silappatikaram is a poetic
r
is of Tamil culture; its varied religions; its town plans and city types; the
rendition with details
t M
mingling of different people; and the arts of dance and music.
ulki
P Improvisation: Page 85-86: TN 11
Q Source: th
Standard History Textbook

www.insightsias.com 70
© Insights Active Learning | All rights reserved - 78168. You may not reproduce, distribute or exploit the contents in any form without
written permission by copyright owner. Copyright infringers may face civil and criminal liability
Total Marks : 200
TEST - 1 (MOCK TEST - 1) Mark Scored : 103.33

63 Consider the following statements.


1. Harsha prohibited the use of animal food in his kingdom and punished those who killed any living
being.
2. Harsha gave away his enormous wealth as gifts to the members of all religious sects during the
Allahabad Conference.

Which of the above is/are correct?


A. 1 only
B. 2 only
C. Both 1 and 2
D. None

Your Answer : B
Correct Answer : C

Answer Justification : 7
2 27
Justification: Statement 1: In his early life, Harsha was a devout Saiva but
5 03later he became an
9 41
ardent Hinayana Buddhist. Hiuen Tsang converted him to Mahayana Buddhism. This led him to
shed eating meat and enforce the same as well. -
o m
Statement 2: Hiuen Tsang mentions in his account about i l .cthe conference held at Allahabad, known
as Prayag. m a
g
a 7@ convened by Harsha once in five years. Harsha gave
r
It was the one among the conferences routinely
is members of all religious sects.
away his enormous wealth as gifts to
it m the

u l k
According to Hiuen Tsang,pHarsha was so lavish that he emptied the treasury and even gave away
-
sra
the clothes and jewels he was wearing. His statement might be one of admiring exaggeration.
i
i
Q Source: Page
k t M116: TN 11 Standard History Textbook
th

l
Pu

www.insightsias.com 71
© Insights Active Learning | All rights reserved - 78168. You may not reproduce, distribute or exploit the contents in any form without
written permission by copyright owner. Copyright infringers may face civil and criminal liability
Total Marks : 200
TEST - 1 (MOCK TEST - 1) Mark Scored : 103.33

64 With reference to the rights and responsibilities of the Attorney General of India, consider the
following:
1. He has right of audience in all courts within the territory of India.
2. He has the right to speak and take part in the proceeding of a joint sitting of the Parliament.
3. He is entitled to vote on crucial national resolutions and motions in Rajya Sabha, excluding
legislations.
4. He can be made a member of any parliamentary committee with a right to discuss and vote.
5. He is not allowed to take up private practice as long as he is an incumbent.

Select the correct answer using the codes below.


A. 1 and 2 only
B. 1, 3 and 4 only
C. 1, 4 and 5 only
D. 2, 3 and 5 only

Your Answer : D
7
Correct Answer : A
2 27
0 3
5
Answer Justification : 41
-9
Justification: He is the first law officer of the governmentoofmIndia and acts as top advocate for
Union Government. He is responsible for giving adviceilto.cPresident/ Union Government upon legal
matters. m a
7 @g
Statement 2 and 3: He has the right to speak
is ra and take part in proceedings of both the houses of
parliament including joint sittings. m But cannot vote in parliament.
i t
lk
umade
Statement 4: He can also be
- p a member of any parliamentary committee but in the committee

srato vote. He has all the powers and privileges that of a member of parliament.
also, he has no power
i
M
Statement 5:itAttorney General of India is not a full time Government servant. He is an advocate of
l k
Pu
the government and is allowed to take up private practice, provided the other party is not the state.
Further, without permission of the government he cannot defend the accused persons in criminal
matters.

Q Source:
http://www.thehindu.com/news/national/attorney-general-rohatgi-will-defend-indias-record-on-rights
-abuses-at-unhrcs-review/article18312468.ece

www.insightsias.com 72
© Insights Active Learning | All rights reserved - 78168. You may not reproduce, distribute or exploit the contents in any form without
written permission by copyright owner. Copyright infringers may face civil and criminal liability
Total Marks : 200
TEST - 1 (MOCK TEST - 1) Mark Scored : 103.33

65 In the context of the rule of South Indian dynasties, Devadhana and Brahmadeya referred to?

A. Pristine land
B. Divine king rule
C. Treasure appropriated to the Gods
D. Land grants to the temples

Your Answer : D
Correct Answer : D

Answer Justification :

Justification: Land tax was the primary source of the government revenue. The Brahmadeya and
Devadhana lands were exempted from tax.

Learning: This is a lateral explanation and deals with aspects other than those mentioned in the
question, because detailing the topic at hand is not of much use. 77 2
2
03
15
94Pallava state was divided into
The Pallavas had a well organized administrative system. The
-
Kottams. The Kottam was administered by officers appointed by the king.
. com
ai l
m
It was also the responsibility of the centralggovernment to provide irrigation facilities to the
lands. 7 @
isra
it m
Traders and artisans such
u lkas carpenters, goldsmiths, washer-men, oil-pressers and weavers
- p
paid taxes to the government.

i sra
M
k it inscriptions throw much light on the village assemblies called sabhas and their
The Pallava
l
u
Pcommittees. They maintained records of all village lands, looked after local affairs and
managed temples.

Q Source: Page 126: TN 11th Standard History Textbook

www.insightsias.com 73
© Insights Active Learning | All rights reserved - 78168. You may not reproduce, distribute or exploit the contents in any form without
written permission by copyright owner. Copyright infringers may face civil and criminal liability
Total Marks : 200
TEST - 1 (MOCK TEST - 1) Mark Scored : 103.33

66 They introduced the art of excavating temples from the rock, and the Dravidian style of temple
architecture began with their rule. They were?

A. Cheras
B. Pallavas
C. Eastern Chalukyas
D. Jaffna

Your Answer : B
Correct Answer : B

Answer Justification :

Learning: Pallava age was a great age of temple building.

It was a gradual evolution starting from the cave temples to monolithic rathas and culminated in
structural temples. 77
3 22
The development of temple architecture under the Pallavas can be seen5in0several stages, which
started with Mahendravarman I introducing rock-cut temples. 9 41
-
m
i l .co
This style of Pallava temples are seen at places like Mandagappattu, Mahendravadi, Mamandur,
Dalavanur, Tiruchirappalli, Vallam, Siyamangalam and
m a Tirukalukkunram.
g
@Textbook
th
7
sra
Q Source: Page 127: TN 11 Standard History
i
k i tm
p ul
a -
is r
M
l k it
Pu

www.insightsias.com 74
© Insights Active Learning | All rights reserved - 78168. You may not reproduce, distribute or exploit the contents in any form without
written permission by copyright owner. Copyright infringers may face civil and criminal liability
Total Marks : 200
TEST - 1 (MOCK TEST - 1) Mark Scored : 103.33

67 In Delhi Sultanate, the Sultan was assisted by a number of departments and officials in his
administration. Consider the following with reference to these departments:
1. Diwani Insha dealt with the correspondence between the ruler and the officials.
2. Diwani Ariz gave grants for the construction and maintenance of mosques, tombs and madrasas.

Which of the above is/are correct?


A. 1 only
B. 2 only
C. Both 1 and 2
D. None

Your Answer : A
Correct Answer : A

Answer Justification :

7
Justification: Statement 1: It is correct.
2 27
3
1 50 by Ariz-i-mumalik. He
Statement 2: The military department was called Diwani Ariz. It was headed
94 department.
was responsible for recruiting the soldiers and administering the military
-
m
Learning: Under the Delhi sultanate, the post of Naib was
i l .cothe most powerful one. The Naib
m a
practically enjoyed all the powers of the Sultan and exercised general control over all the
departments.
7 @g
isra
Next to him was the Wazir iwho
k tmwas heading the finance department called Diwani Wizarat.
p ul
a -
Diwani Rasalat
i sr was the department of religious affairs. It was headed by chief Sadr. Grants
were made
t M by this department for religious construction.
lk i
P u
Q Source: Page 192: TN 11th Standard History Textbook

www.insightsias.com 75
© Insights Active Learning | All rights reserved - 78168. You may not reproduce, distribute or exploit the contents in any form without
written permission by copyright owner. Copyright infringers may face civil and criminal liability
Total Marks : 200
TEST - 1 (MOCK TEST - 1) Mark Scored : 103.33

68 In the context of Ancient India, Amogavarsha’s work Kavirajamarga is significant because?

A. It was the first poetic work in Kannada language.


B. The largest repository of royal assets was commissioned in this work.
C. All artists of the empire contributed to this work.
D. It was a prose written in the middle of a battle ground.

Your Answer : B
Correct Answer : A

Answer Justification :

Learning: The Kannada literature saw its beginning during the period of the Rashtrakutas.
Amogavarsha’s Kavirajamarga was the first poetic work in Kannada language.

The name literally means "Royal Path for Poets" and was written as a guide book for poets and
scholars (Kavishiksha). 77 2
2
Q Source: Page 137: TN 11th Standard History Textbook 503
1
- 94
. com
a i l
m
7 @g
isra
it m
p ulk
a -
i sr
M
l k it
Pu

www.insightsias.com 76
© Insights Active Learning | All rights reserved - 78168. You may not reproduce, distribute or exploit the contents in any form without
written permission by copyright owner. Copyright infringers may face civil and criminal liability
Total Marks : 200
TEST - 1 (MOCK TEST - 1) Mark Scored : 103.33

69 Several accords have been signed between the Government of India (GoI) and different groups to
bring peace and prosperity to the North-eastern states. Consider the following with reference to
them.
1. Mizo accord promised statehood to Mizoram.
2. Assam Accord was reached between GoI and ULFA calming the Assam agitation.
3. Nagaland peace accord between GoI and the National Socialist Council of Nagaland (NSCN) was
signed to end the Naga insurgency.

Select the correct answer using the codes below.


A. 1 and 3 only
B. 3 only
C. 2 and 3 only
D. 1 and 2 only

Your Answer : A
Correct Answer : A
7
2 27
Answer Justification :
0 3
5
41
9 Front (MNF) fighters
Justification: Statement 1: Under the Mizo accord, the Mizo National
-
mgiving full-fledged state status to
surrendered en masse and government fulfilled its promise by
o
Mizoram, along with its own High Court. l. c i
m a
@g
s r a7language. Further, Mizo religious and social customs
Mizo was notified as an official Indian
i
were promised Constitutional protection.
m
l k it
p u
As per the accord, -the MNF agreed to forgo all contacts with other insurgent groups in the
a
isr Accord is considered to be the most successful agreement with insurgents.
Northeast. Mizo
M
l k it
u
P 2: ULFA is a banned organization. So, 2 is wrong.
Statement

The accord brought an end to the Assam Agitation and paved the way for the leaders of the
agitation to form a political party and form a government in the state of Assam soon after. However,
not all promises have been fulfilled that remains a bone of contention.

Statement 3: This article explains the history and issues with Naga insurgency very clearly and
chronologically: A must read – This is an important issue.
http://indianexpress.com/article/explained/simply-put-towards-accord-step-by-step/

Q Source:
http://www.thehindu.com/news/national/mizoram-celebrates-31st-anniversary-of-mizo-peace-accord/
article19187194.ece

www.insightsias.com 77
© Insights Active Learning | All rights reserved - 78168. You may not reproduce, distribute or exploit the contents in any form without
written permission by copyright owner. Copyright infringers may face civil and criminal liability
Total Marks : 200
TEST - 1 (MOCK TEST - 1) Mark Scored : 103.33

70 The system of village autonomy with sabhas and their committees developed through the ages and
reached its culmination during the Chola rule. A village was divided in several wards, what was/were
the qualifications to become a ward member?
1. A woman could not become a ward member.
2. Only those who have completed seventy years of age
3. Have knowledge of Vedas
4. Owns a residence

Select the correct answer using the codes below.


A. 3 and 4 only
B. 1, 2 and 3 only
C. 1 and 4 only
D. 1, 2, 3 and 4

Your Answer : A
Correct Answer : A
7
2 27
Answer Justification :
0 3
5
41
Learning: That village was divided into thirty wards and each was9to nominate its members to the
-
village council. m
i l .co
The qualifications to become a ward member were:
m a
@ g
7
Ownership of at least one fourthis
ra
veli of land.
itm
u l k
Own residence. - p
sr a
M i
t years and below seventy years of age.
kithirty
Above
l
Pu
Knowledge of Vedas.

However, certain norms of disqualification were also mentioned in the inscriptions. They were:

Those who had been members of the committees for the past three years.

Those who had failed to submit accounts as committee members.

Those who had committed sins.

Those who had stolen the property of others

Two inscriptions
www.insightsias.com belonging to the period of Parantaka I found at Uttiramerur provide details of the78
© Insights Active Learning | All rights reserved - 78168. You may not reproduce, distribute or exploit the contents in any form without
written permission by copyright owner. Copyright infringers may face civil and criminal liability
Total Marks : 200
TEST - 1 (MOCK TEST - 1) Mark Scored : 103.33

formation and functions of village councils.

Q Source: Page 148: TN 11th Standard History Textbook

7
2 27
0 3
5
41
-9
. com
ai l
gm
7 @
isra
i t m
p ulk
a -
i sr
M
l k it
Pu

www.insightsias.com 79
© Insights Active Learning | All rights reserved - 78168. You may not reproduce, distribute or exploit the contents in any form without
written permission by copyright owner. Copyright infringers may face civil and criminal liability
Total Marks : 200
TEST - 1 (MOCK TEST - 1) Mark Scored : 103.33

71 In the ancient traditions, traders’ voyages often refer which of these lands to Suvarnabhumi (the land
of gold)?

A. Countries of the East Asia


B. African nations
C. Sri Lankan islands
D. European countries

Your Answer : B
Correct Answer : A

Answer Justification :

Learning: The east coast of India is studded with numerous ports and Indians undertook frequent
voyages to these lands.

2 77
3
Also, Indians began to colonize the East Asia in the Gupta period. It was2further encouraged
by the Pallavas. 5 0
4 1
-9
The Indian colonists established great kingdoms (with
. com
Indian names) and some of them lasted
for more than a thousand years. a i l
gm
7 @
sra century, Indian culture dominated this region.
Till the arrival of Islam in the fifteenth
i
m
lkit
pthuStandard History Textbook
Q Source: Page 156: TN 11
-
a
M isr
l k it
Pu

www.insightsias.com 80
© Insights Active Learning | All rights reserved - 78168. You may not reproduce, distribute or exploit the contents in any form without
written permission by copyright owner. Copyright infringers may face civil and criminal liability
Total Marks : 200
TEST - 1 (MOCK TEST - 1) Mark Scored : 103.33

72 The system of four ashrams is advocated in

A. Buddhism
B. Jainism
C. Lokayatas
D. Brahmanism

Your Answer : D
Correct Answer : D

Answer Justification :

Learning: Around the time when Jainism and Buddhism were becoming popular, brahmins
developed the system of ashramas.

Ashramas refer to stage of life: brahmacharya, grihastha, vanaprastha and7 7


samnyasa.
2 2
5 03
41 lives and study the Vedas
Brahmin, kshatriya and vaishya men were expected to lead simple
9
during the early years of their life (brahmacharya). -
. com
ai l
m (grihastha).
Then they had to marry and live as householders
g
@
sr a7
m i and meditate (vanaprastha). Finally, they had to give up
Then they had to live in the forest
t
lki
everything and become samnyasins.
u
- p
r a
isashramas allowed men to spend some part of their lives in meditation.
The system of
M
l k it
u
PGenerally, women were not allowed to study the Vedas, and they had to follow the ashramas
chosen by their husbands.

Q Source: Page 173: TN 11th Standard History Textbook

www.insightsias.com 81
© Insights Active Learning | All rights reserved - 78168. You may not reproduce, distribute or exploit the contents in any form without
written permission by copyright owner. Copyright infringers may face civil and criminal liability
Total Marks : 200
TEST - 1 (MOCK TEST - 1) Mark Scored : 103.33

73 ‘Petya’ recently seen in news was a/an


#40166

A. Laser beaming innovation


B. Form of cyber-attack
C. Alternate to fossil fuels
D. Emergency helpline launched by PETA

Your Answer : B
Correct Answer : B

Answer Justification :

Learning: It is a ransomware, similar to the Wannacry ransomware.

In India, this ransomware crippled the operations at one of the terminals of the Jawaharlal Nehru
7 cargo and
Port Trust and impacted the external trade by affecting the systems dealing with7the
2 2
ships. 3
1 50
94 the victim's data or
Ransomware is a type of malicious software that threatens to publish
-
om
perpetually block access to it unless a ransom is paid.
l . c
Q Source:
m ai
@g
https://www.theguardian.com/technology/2017/jun/27/petya-ransomware-cyber-attack-who-what-wh
7
sra
y-how
i
k i tm
p ul
a -
isr
M
l k it
Pu

www.insightsias.com 82
© Insights Active Learning | All rights reserved - 78168. You may not reproduce, distribute or exploit the contents in any form without
written permission by copyright owner. Copyright infringers may face civil and criminal liability
Total Marks : 200
TEST - 1 (MOCK TEST - 1) Mark Scored : 103.33

74 In 1329-30 Muhammad bin Tughlaq introduced a token copper currency in place of silver, but banned
it later and promised to exchange silver coins for these copper coins. This was done because

A. Copper was scarce in the empire, but had huge demand in the world.
B. Token currency was trampled and disrespected by people.
C. Goldsmiths began to forge the token coins on a large scale
D. Treasury became empty with issue of token coins.

Your Answer : C
Correct Answer : C

Answer Justification :

Learning: There was a shortage of silver through out the world in the fourteenth century.

Kublai Khan issued paper money in China. In the same manner, Muhammad 77bin Tughlaq
issued copper coins at par with the value of the silver tanka coins. 22 3
0
15
94
But he was not able to prevent forging the new coins. The -goldsmiths began to forge the
token coins on a large scale. Soon the new coins were
. com
not accepted in the markets due to its
widescale duplication and thus credibility. a i l
gm
7 @
Finally, Muhammad bin Tughlaq s
i ra the circulation of token currency and promised to
stopped
exchange silver coins for thetm
copper coins.
lk i
- pu
ra
Many people exchanged
is
the new coins but the treasury became empty.
M
l k it
Pu
According the Barani, the heap of copper coins remained lying on roadside in Tughlaqabad.

Q Source: Page 184: TN 11th Standard History Textbook

www.insightsias.com 83
© Insights Active Learning | All rights reserved - 78168. You may not reproduce, distribute or exploit the contents in any form without
written permission by copyright owner. Copyright infringers may face civil and criminal liability
Total Marks : 200
TEST - 1 (MOCK TEST - 1) Mark Scored : 103.33

75 Writing in the journal Young India in 1931, who said, "I cannot possibly bear the idea that a man who
has got wealth should get the vote but a man who has got character but no wealth or literacy should
have no vote…. for the crime of being a poor man…"?

A. Jawahar Lal Nehru


B. M.K. Gandhi
C. Rajendra Prasad
D. Maulana Abul Kalam Azad

Your Answer : B
Correct Answer : B

Answer Justification :

Learning: Young India, Harijan and Hind Swaraj were some of the most famous works of Gandhi.

7
27
Young India was a weekshed - a weekly paper or journal.
3 2
50 the use of nonviolence
He used Young India to spread his unique ideology and thoughts regarding
1
94and plan for India's eventual
in organising movements and to urge readers to consider, organise,
-
independence from Britain.
. com
l
i League of America.
The journal was reprinted in USA by the India HomeaRule
m
Q Source: Improvisation: Page 33: NCERT 7 @g VI Social and Political Life - I
Class
isra
k i tm
p ul
a -
isr
M
l k it
Pu

www.insightsias.com 84
© Insights Active Learning | All rights reserved - 78168. You may not reproduce, distribute or exploit the contents in any form without
written permission by copyright owner. Copyright infringers may face civil and criminal liability
Total Marks : 200
TEST - 1 (MOCK TEST - 1) Mark Scored : 103.33

76 In the context of Pallava’s history, what was Dakshinachitra?

A. A compendium of temples
B. A spiritual treatise on Pallava practices
C. An autobiography of a Pallava ruler
D. An instruction manual for painters

Your Answer : A
Correct Answer : D

Answer Justification :

Learning: Mahendravarman, one of the greatest of the rulers of the Pallava dynasty, was found of
painting and is said to have commissioned a scholar at his court to pen down an instruction manual
for painters, which is a treatise on south Indian paintings called Dakshinachitra.

7
Unfortunately, it has been lost in the course of time.
2 27
0 3
15 new fangled ideas).
Mahendravarman I gave himself the title of “Vchitrachitta” (the man with
4
th -9
om
Q Source: Page 129: TN 11 Standard History Textbook
l . c
m ai
@ g
7
isra
it m
p ulk
a -
i sr
M
l k it
Pu

www.insightsias.com 85
© Insights Active Learning | All rights reserved - 78168. You may not reproduce, distribute or exploit the contents in any form without
written permission by copyright owner. Copyright infringers may face civil and criminal liability
Total Marks : 200
TEST - 1 (MOCK TEST - 1) Mark Scored : 103.33

77 In medieval India, takkavi loans were given in order to

A. Improve agriculture
B. Build religious shrines
C. Construct flood saving embankments in cities
D. Serve sovereign debt

Your Answer : A
Correct Answer : A

Answer Justification :

Learning: Takkavi loans were a measure of reprieve taken after few suppressing moves of Tughlaq.

In order to overcome financial difficulties in his empire, Muhammad bin Tughlaq increased
the land revenue on the farmers of Doab (land between Ganges and Yamuna 77rivers). 2
2
5 03
It was an excessive and arbitrary step on the farmers. A severe
9 41 famine was also ravaging that
- The revolts were crushed.
region at that time. It had resulted in a serious peasant revolts.
. com
ai l
m relief measures and the promotion of
However, the Sultan realized later that adequate
g
7@problem.
agriculture were the real solution to the
isra
t m
i takkavi loans (loans for cultivation) were given to the farmers
He launched a scheme by
u lkwhich
p agriculture.
to buy seed and to extend
-
a
M isr
it 193: TN 11 Standard History Textbook
Q Source: Page
th

Pulk

www.insightsias.com 86
© Insights Active Learning | All rights reserved - 78168. You may not reproduce, distribute or exploit the contents in any form without
written permission by copyright owner. Copyright infringers may face civil and criminal liability
Total Marks : 200
TEST - 1 (MOCK TEST - 1) Mark Scored : 103.33

78 Consider the following statements.


1. Sankara denounced Hindu revivalist movement citing them as going against rationality and
modernity.
2. Ramanuja advocated prabattimarga or the path of self-surrender to God for salvation.

Which of the above is/are correct?


A. 1 only
B. 2 only
C. Both 1 and 2
D. None

Your Answer : B
Correct Answer : B

Answer Justification :

7
Learning: Statement 1: In the ninth century Sankara started a Hindu revivalist2movement giving a
7
new orientation to Hinduism. So, 1 is wrong. 3 2
5 0
41
9 to the common man.
However, his doctrine of Advaita or Monism was too abstract to appeal
-
m
Moreover, there was a reaction against the Advaita concept
i l .co of Nirgunabrahman (God without
m a
attributes) with the emergence of the idea of Sagunabrahman (God with attributes).
g
Statement 2: In the twelfth century, Ramanuja,a 7@preached Visishtadvaita.
is r
m
According to him God u
kit
is lSagunabrahman. The creative process and all the objects in creation
- p as was held by Sankaracharya.
are real but not illusory
i sra
k it MGod, soul, matter are real. But God is inner substance and the rest are his
ul
Therefore,
Pattributes.

He invited the downtrodden to Vaishnavism

Q Source: Page 202: TN 11th Standard History Textbook

www.insightsias.com 87
© Insights Active Learning | All rights reserved - 78168. You may not reproduce, distribute or exploit the contents in any form without
written permission by copyright owner. Copyright infringers may face civil and criminal liability
Total Marks : 200
TEST - 1 (MOCK TEST - 1) Mark Scored : 103.33

79 Consider the following statements.


1. A Supernova helps us know about the Universe since they are the longest existing phenomenon in
space.
2. Pulsars are de-magnetized neutron stars that neither rotate nor emit electromagnetic radiation.
3. Black hole refers to the remains of a supergiant star that has collapsed into itself.
4. The effects of microgravity can be seen when astronauts and objects float in space.

Select the correct answer using the codes below.


A. 1 and 3 only
B. 3 and 4 only
C. 2, 3 and 4 only
D. 1, 2 and 4 only

Your Answer : A
Correct Answer : B
7
Answer Justification :
2 27
0 3
15 black holes, pulsars and
Background: China has launched its first X-ray space telescope to study
4
gamma-ray bursts. -9
om
il.cthere is a change in the core, or centre, of
Justification: Statement 1: A supernova happens where
a
a star. m
7 @g
A supernova burns for only a short period
israof time, but it can tell scientists a lot about the universe.
i m
tscientists
One kind of supernova has shown
u l k that we live in an expanding universe, one that is
p rate.
growing at an ever increasing
-
a
srare
M i
Statement 2: Pulsars highly magnetized, rotating neutron stars that emit a beam of

l k it radiation.
electromagnetic
Pu
They are spherical, compact objects that are about the size of a large city but contain more
mass than that of the sun.

Scientists are using pulsars to study extreme states of matter, search for planets beyond
Earth’s solar system and measure cosmic distance.

Statement 3: A black hole cannot be seen because of the strong gravity that is pulling all of the light
into the black hole's centre.

However, scientists can see the effects of its strong gravity on the stars and gases around it.

If a star is orbiting a certain point in space, scientists can study the star's motion to find out if
it is orbiting a black hole.

www.insightsias.com 88
© Insights Active Learning | All rights reserved - 78168. You may not reproduce, distribute or exploit the contents in any form without
written permission by copyright owner. Copyright infringers may face civil and criminal liability
Total Marks : 200
TEST - 1 (MOCK TEST - 1) Mark Scored : 103.33

Statement 4: Microgravity is the condition in which people or objects appear to be weightless. This
occurs when gravitation force around is weak (but not zero) or when objects are in a state of free
fall (as in a satellite).

For example, astronauts can move equipment weighing hundreds of pounds with their fingertips
due to microgravity.

Q Source: NASA Website and China’s telescope launch

7
2 27
0 3
5
41
-9
. com
ai l
gm
7 @
isra
i t m
p ulk
a -
i sr
M
l k it
Pu

www.insightsias.com 89
© Insights Active Learning | All rights reserved - 78168. You may not reproduce, distribute or exploit the contents in any form without
written permission by copyright owner. Copyright infringers may face civil and criminal liability
Total Marks : 200
TEST - 1 (MOCK TEST - 1) Mark Scored : 103.33

80 The highest court of appeal in the Vijayanagar Empire was

A. Gram Sabha
B. Mantri Parishad
C. Qazi’s Court
D. The King

Your Answer : D
Correct Answer : D

Answer Justification :

Learning: The administration under the Vijayanagar Empire was organized in the way that the king
enjoyed absolute authority in executive, judicial and legislative matters. He was the highest court of
appeal.

77 word of the
The king was assisted by a council of ministers in his day to day administration. But,
2
king was final. 3 2
1 50
4
-9
Moreover, the succession to the throne was on the principle of hereditary.
m
Q Source: Page 212: TN 11 Standard History Textbook.co
th

ai l
m
7 @g
isra
k i tm
p ul
a -
isr
M
l k it
Pu

www.insightsias.com 90
© Insights Active Learning | All rights reserved - 78168. You may not reproduce, distribute or exploit the contents in any form without
written permission by copyright owner. Copyright infringers may face civil and criminal liability
Total Marks : 200
TEST - 1 (MOCK TEST - 1) Mark Scored : 103.33

81 The beginning of agriculture can be ascribed most suitably to which of these times?

A. 2,500 years ago with the onset of Magadha empire


B. 4,700 years ago with the appearance of the first cities on the Indus
C. Nearly 8,000-10,000 years ago
D. About 25,000 years ago in the first Penistone glacial period

Your Answer : A
Correct Answer : C

Answer Justification :

Justification & Learning: Option D: People evolved into their current form some 200,000 years
ago (see human evolution), yet they did not begin to engage in agriculture until about
15,000–10,000 years before the present.

7
Wild grains were collected and eaten from at least 20,000 BC. So, D is wrong.
2 27
3
1
Option C: The first agriculture appears to have developed at the closing5of0the last Pleistocene
glacial period, or Ice Age (about 11,700 years ago).
- 94
m
oemmer wheat, einkorn wheat, hulled
From around 9,500 BC, the Neolithic founder crops such.cas
i l
a domesticated in China between 11,500 and
barley, peas etc were cultivated in the Levant. Rice was
m
6,200 BC.
7 @g
Q Source: Page 10: NCERT Class VI Our israPasts - I
k i tm
p ul
a -
isr
M
l k it
Pu

www.insightsias.com 91
© Insights Active Learning | All rights reserved - 78168. You may not reproduce, distribute or exploit the contents in any form without
written permission by copyright owner. Copyright infringers may face civil and criminal liability
Total Marks : 200
TEST - 1 (MOCK TEST - 1) Mark Scored : 103.33

82 With reference to the International Tribunal for the Law of the Sea (ITLOS), consider the following:
1. It is an organ of the United Nations adjudicating disputes arising out of the interpretation and
application of the UN Convention on the Law of the Seas (UNCLOS).
2. Each state party of the ITLOS nominates a representative to become a member of ITLOS.

Which of the above is/are correct?


A. 1 only
B. 2 only
C. Both 1 and 2
D. None

Your Answer : A
Correct Answer : D

Answer Justification :
7
2 27
Justification: Statement 1: The Tribunal is an independent judicial body but maintains close links
with the United Nations. 03
4 15
The Tribunal and the UN have entered into agreement concerning - 9cooperation and relationship.
. com
UN has granted the Tribunal observer status in the General
a i l Assembly.

g m
Statement 2: The Tribunal is composed of 21 independent members, elected from among persons
7 @
sra
enjoying the highest reputation for fairness and integrity and of recognized competence in the field
of the law of the sea. i
k i tm
p ul
a - Neeru Chadha has become the first Indian women to elected to the
Recently, law expert
r
isTribunal for the Law of the Seas (ITLOS).
International
M
l k it
u has also earned a distinction of becoming the second Indian woman to occupy a top
PNeeru
position at the United Nations after Lakshmi Pandit who served as the President of the United
Nations General Assembly.

Q Source:
http://www.thehindu.com/news/international/neeru-chadha-becomes-1st-indian-woman-as-member-o
f-itlos/article19053500.ece

www.insightsias.com 92
© Insights Active Learning | All rights reserved - 78168. You may not reproduce, distribute or exploit the contents in any form without
written permission by copyright owner. Copyright infringers may face civil and criminal liability
Total Marks : 200
TEST - 1 (MOCK TEST - 1) Mark Scored : 103.33

83 This archaeological site consist of a number of low archaeological mounds created by generations of
superimposed mudbrick structures. Located close to the mouth of the Bolan Pass, it was abandoned
by the time of the emergence of the literate urbanised phase of the Indus Civilisation. An entry in the
UNESCO tentative list it is?

A. Lothal
B. Mehrgarh
C. Inamgaon
D. Surkotada

Your Answer : B
Correct Answer : B

Answer Justification :

Learning: It was a small farming village. It is one of the earliest sites with evidence of farming and
herding in South Asia. 77
3 22
1 50
Mehrgarh is now seen as a precursor to the Indus Valley Civilization, displaying the whole sequence
4
from earliest settlement and the start of agriculture, to the mature9Harappan Civilisation (based on
the above Q description of site abandonment). -
m
i l .co
m a
Most of its archaeological deposits are buried deep beneath accumulations of alluvium.

Q Source: Improvisation: Page 29 (image):7 @g Class VI Our Pasts - I


NCERT
isra
k i tm
p ul
a -
isr
M
l k it
Pu

www.insightsias.com 93
© Insights Active Learning | All rights reserved - 78168. You may not reproduce, distribute or exploit the contents in any form without
written permission by copyright owner. Copyright infringers may face civil and criminal liability
Total Marks : 200
TEST - 1 (MOCK TEST - 1) Mark Scored : 103.33

84 Integrated Child Development Services (ICDS) Scheme has a bearing on


1. Infant Mortality rate (IMR)
2. Maternal Mortality rate (MMR)
3. Reduction of malnutrition in children
4. Pre-school non-formal education

Select the correct answer using the codes below.


A. 1 and 3 only
B. 2 and 4 only
C. 1, 2 and 3 only
D. 1, 2, 3 and 4

Your Answer : D
Correct Answer : D

Answer Justification :
7
2 27
5 03
Justification: It is a centrally sponsored scheme implemented by the states/UTs. The Scheme was
launched in 1975.
9 41
-
Statement 1 and 3: The scheme has the following objectives:m
i l .co
m a
g of children in the age-group 0-6 years; this leads
to improve the nutritional and health status
@
7
sra
to an improvement in IMR.
i
k i tm
p ul
to reduce mortality, morbidity, malnutrition and school dropout;
a -
is r
to promote M policy coordination and implementation for promoting child development;
l k it
Pu
to lay an adequate foundation for the psychological, physical and social well-being of the
children etc.

Statement 2 and 4: Under the scheme, the services such as Supplementary nutrition; Pre-school
non-formal education; nutrition and health education, immunization, health check-ups; and Referral
services through Anganwadi Centres for children below 6 years of age as well as to pregnant
women& lactating mothers are offered. These improve MMR.

Learning: Infant mortality rate (IMR) is the number of deaths per 1,000 live births of children
under one year of age. It is presently around 40 in India.

The Maternal mortality rate (MMR) is the annual number of female deaths per 100,000 live births
from any cause related to or aggravated by pregnancy or its management (excluding accidental or
incidental causes). The recent World Bank data puts the MMR for India reported in 2015 at 174.

Q Source: CRY report on children in India

www.insightsias.com 94
© Insights Active Learning | All rights reserved - 78168. You may not reproduce, distribute or exploit the contents in any form without
written permission by copyright owner. Copyright infringers may face civil and criminal liability
Total Marks : 200
TEST - 1 (MOCK TEST - 1) Mark Scored : 103.33

85 The river(s) that do NOT find mention in Rigveda is?

A. Ganga and Yamuna


B. Indus and its other tributaries
C. Saraswati
D. All of the above find mention.

Your Answer : B
Correct Answer : D

Answer Justification :

Learning: Option B: There is a hymn in the form of a dialogue between sage Vishvamitra, and two
rivers, (Beas and Sutlej) that were worshipped as goddesses.

Option C: The most prominent river of the Rigveda is the Sarasvati, next to the Indus.
7
2 27
Option A: The Ganga and Yamuna are named only once.
0 3
5
41
-9
Q Source: Page 45: NCERT Class VI Our Pasts - I

. com
ai l
gm
7 @
isra
it m
p ulk
a -
i sr
M
l k it
Pu

www.insightsias.com 95
© Insights Active Learning | All rights reserved - 78168. You may not reproduce, distribute or exploit the contents in any form without
written permission by copyright owner. Copyright infringers may face civil and criminal liability
Total Marks : 200
TEST - 1 (MOCK TEST - 1) Mark Scored : 103.33

86 There are plenty of source materials to reconstruct the history of the Gupta period. Some of the
renowned material, in this context, includes
1. Puranas throw light on the royal genealogy of the Gupta kings.
2. Devichandraguptam and Mudhrakshasam provide information regarding the rise of the Guptas
3. Meherauli Iron Pillar Inscription mention their conquests.
4. Fahien’s writings gives an account of the social, economic and religious condition of the empire.

Select the correct answer using the codes below.


A. 1 and 4 only
B. 2, 3 and 4 only
C. 1 and 3 only
D. 1, 2, 3 and 4

Your Answer : B
Correct Answer : D
7
Answer Justification :
2 27
0 3
5
Learning: S1 and S2 mention commonly known sources.
9 41
Statement 3: The Meherauli Iron Pillar inscription mentions m
-
the extensive conquests of
co
Chandragupta I. He is considered to be the founder of the
a il. Gupta era which starts with his accession
in A.D. 320.
gm
a 7@Samudragupta defeated twelve rulers in his South
The Allahabad Pillar inscription mentions
is r that
Indian Expedition.
it m
l k
Statement 4: We will coverpauseparate question on Fahien’s accounts. It is important to note that his
a
main interest of visiting -India was religious.
is r
M
it 96-97: TN 11 Standard History Textbook
th
Q Source: Page
l k
Pu

www.insightsias.com 96
© Insights Active Learning | All rights reserved - 78168. You may not reproduce, distribute or exploit the contents in any form without
written permission by copyright owner. Copyright infringers may face civil and criminal liability
Total Marks : 200
TEST - 1 (MOCK TEST - 1) Mark Scored : 103.33

87 Buddhist Viharas are

A. A monument to meditate and chant for the entire Sangha


B. Permanent shelters or monasteries for dwelling and rest
C. Divine extensions of stupas
D. Sites containing relics of Buddha

Your Answer : B
Correct Answer : B

Answer Justification :

Learning: To begin with, both Jaina and Buddhist monks went from place to place throughout the
year, teaching people.

277
The only time they stayed in one place was during the rainy season, when it was very difficult
to travel. 3 2
5 0
9 41
Then, their supporters built temporary shelters for them in- gardens, or they lived in natural
m
caves in hilly areas. coil .
m a
@ g
7 monks and nuns, and they themselves, felt the need
As time went on, many supporters of the
ramonasteries were built.
for more permanent shelters andsso
i
m
u lkit
pviharas.
These were known as
-
a
M isr
it 71: NCERT Class VI Our Pasts - I
Q Source: Page
Pulk

www.insightsias.com 97
© Insights Active Learning | All rights reserved - 78168. You may not reproduce, distribute or exploit the contents in any form without
written permission by copyright owner. Copyright infringers may face civil and criminal liability
Total Marks : 200
TEST - 1 (MOCK TEST - 1) Mark Scored : 103.33

88 PRAGATI is a unique integrating and interactive platform to


1. Address the grievances of citizens
2. Monitoring and reviewing important government projects

Which of the above is/are correct?


A. 1 only
B. 2 only
C. Both 1 and 2
D. None

Your Answer : A
Correct Answer : C

Answer Justification :

Justification: PRAGATI stands for Pro-Active Governance And Timely Implementation.


7
2 27
Key features of the PRAGATI application are as follows:
0 3
5
9 41
- and Chief Secretaries of the
m
It is a three-tier system (PMO, Union Government Secretaries,
States); .co il
m a
@ g
s r a7
Prime Minister will hold a monthly programme where he will interact with the Government of

m i
India Secretaries, and Chief Secretaries through Video-conferencing enabled by data and geo-
informatics visuals; it
lk
u
-p
sra before the PM are picked up from the available database regarding
Issues to be flagged
i
M
Public Grievances, on-going Programmes and pending Projects;
l k it
Pu
It will also take into consideration various correspondences to PM’s office by the common
people or from high dignitaries of States and/or developers of public projects;

Q Source: PIB AKT/NT: Release ID :167366

www.insightsias.com 98
© Insights Active Learning | All rights reserved - 78168. You may not reproduce, distribute or exploit the contents in any form without
written permission by copyright owner. Copyright infringers may face civil and criminal liability
Total Marks : 200
TEST - 1 (MOCK TEST - 1) Mark Scored : 103.33

89 In ancient Southern India, kadaisiyar and adimai were used to refer to

A. Ordinary ploughmen
B. Landless labourers, including slaves
C. Tax officials
D. Large landowners

Your Answer : C
Correct Answer : B

Answer Justification :

Learning: Society was segregated even in ancient India.

There were at least three different kinds of people living in most villages in the southern and
northern parts of the ancient Indian subcontinent.
2 77
2
03and adimai.
In the Tamil region, large landowners were known as vellalar, ordinary ploughmen were known as
5
41
uzhavar, and landless labourers, including slaves, were known as kadaisiyar
9
- political position and assets held.
m
In the northern region, this distinction was made in terms of land,

i l .co
Q Source: Page 88: NCERT Class VI Our Pasts - I a
gm
a 7@
is r
it m
l k
- pu
i sra
k itM
l
Pu

www.insightsias.com 99
© Insights Active Learning | All rights reserved - 78168. You may not reproduce, distribute or exploit the contents in any form without
written permission by copyright owner. Copyright infringers may face civil and criminal liability
Total Marks : 200
TEST - 1 (MOCK TEST - 1) Mark Scored : 103.33

90 Popularly known Jatakas were

A. Stories of previous lives of the Buddha


B. Objects of concentration
C. Donation passed from student to teacher
D. Arahants who sacrificed their lives for the service of Sangha

Your Answer : A
Correct Answer : A

Answer Justification :

Learning: They are a voluminous body of literature native to India concerning the previous births
of Gautama Buddha in both human and animal form.

77 in whatever
The future Buddha may appear as a king, an outcast, a god, an elephant—but,
2
form, he exhibits some virtue that the tale thereby inculcates. 3 2
5 0
9 41
In Theravada Buddhism, the Jatakas are a textual division -of the Pali Canon, included in the
m
Khuddaka Nikaya of the Sutta Pitaka. co il .
m a
@ g
7
The term Jataka may also refer to a traditional
a
commentary on this book. Q Source: Page 89:
NCERT Class VI Our Pasts – I s r i
m
u lkit
-p
i sra
M
l k it
Pu

www.insightsias.com 100
© Insights Active Learning | All rights reserved - 78168. You may not reproduce, distribute or exploit the contents in any form without
written permission by copyright owner. Copyright infringers may face civil and criminal liability
Total Marks : 200
TEST - 1 (MOCK TEST - 1) Mark Scored : 103.33

91 Consider the following statements.


1. The Anguttara Nikaya, a part of Abhidhamma Pitaka, contains thematically linked discourses of
Buddha.
2. The Majjhima Nikaya, a segment of Vinaya Pitaka, contains the smallest discourses given by the
Buddha to his close disciples.

Which of the above is/are correct?


A. 1 only
B. 2 only
C. Both 1 and 2
D. None

Your Answer :
Correct Answer : D

Answer Justification :
7
2 27
5 03
Justification: Both are a part of Sutta Pitaka. The word Nikaya is most commonly used in reference
to the Buddhist texts of the Sutta Piṭaka.
9 41
-
Statement 1: This is incorrect because Abhidhamma Pitaka isma detailed scholastic reworking of
i l .co discourses by the Buddha himself.
material appearing in the Suttas, not a collection of original
m a
Statement 2: This is incorrect because Vinaya@ g contains rules for the Sangha, it isn’t known
pitaka
7
for hosting discourses.
isra
Q Source: Improvisation: Pagek47: i tm th

u l TN 11 Standard History Textbook


- p
i sra
k itM
l
Pu

www.insightsias.com 101
© Insights Active Learning | All rights reserved - 78168. You may not reproduce, distribute or exploit the contents in any form without
written permission by copyright owner. Copyright infringers may face civil and criminal liability
Total Marks : 200
TEST - 1 (MOCK TEST - 1) Mark Scored : 103.33

92 Changthangi goat of Northern Himalayan region is known for yielding

A. Pashmina wool
B. Shahtoosh
C. Himayalan Yew
D. Pitas and Humas

Your Answer : A
Correct Answer : A

Answer Justification :

Learning: They are a breed of goat inhabiting the plateaus in Tibet and neighbouring areas of
Ladakh in Jammu & Kashmir.

They are raised for ultra-fine cashmere wool Pashmina, which literally translates to "Soft Gold" in
Kashmiri. 77 2
2
03
5 Leh and Ladakh region.
The Changthangi goats have revitalized the poor economy of Changthang,
4 1
Q Source: Page 9: NCERT Class VI Social and Political Life - I -9
. com
ai l
gm
7 @
isra
it m
p ulk
a -
i sr
M
l k it
Pu

www.insightsias.com 102
© Insights Active Learning | All rights reserved - 78168. You may not reproduce, distribute or exploit the contents in any form without
written permission by copyright owner. Copyright infringers may face civil and criminal liability
Total Marks : 200
TEST - 1 (MOCK TEST - 1) Mark Scored : 103.33

93 Consider the following about the role and contributions of B.R. Ambedkar in modern India:
1. He inspired the Dalit Buddhist Movement.
2. He was Independent India's first law minister.
3. He served as a Governor of Reserve Bank of India.

Select the correct answer using the codes below.


A. 1 and 2 only
B. 2 and 3 only
C. 3 only
D. 1 and 3 only

Your Answer : A
Correct Answer : A

Answer Justification :
7
27 freedom for
Justification: Statement 1: He spent his life advocating political rights and social
2
3
Dalits. In 1956 he converted to Buddhism, initiating mass conversions of Dalits.
0
5
41
- 9 that Ambedkar presented to
Statement 3: The Reserve Bank of India (RBI), was based on the ideas
the Hilton Young Commission. He, however, did not serve asmits Governor.
i l .co
m a
Ambedkar was the first Indian to pursue agdoctorate in economics abroad. He argued that
@
a7 could enhance the Indian economy.
industrialisation and agricultural growth
sr
m i
kit
ul 20: NCERT Class VI Social and Political Life - I
Q Source: Improvisation: Page
- p
sr a
M i
l k it
Pu

www.insightsias.com 103
© Insights Active Learning | All rights reserved - 78168. You may not reproduce, distribute or exploit the contents in any form without
written permission by copyright owner. Copyright infringers may face civil and criminal liability
Total Marks : 200
TEST - 1 (MOCK TEST - 1) Mark Scored : 103.33

94 SATH - ‘Sustainable Action for Transforming Human capital’ initiative of NITI Aayog focuses on

A. Creating ‘role model’ states for health systems


B. Revamping higher education
C. Building workforce for the future in areas of cutting edge technology
D. Eradicating manual scavenging

Your Answer : B
Correct Answer : A

Answer Justification :

Learning: This was launched with the State Governments for furthering the agenda of cooperative
federalism.

77‘role model’
NITI Aayog under the SATH initiative plans to identify and build three future
2
states for health systems. 3 2
5 0
9 41
After identification of three states, NITI Aayog will work in- close collaboration with the state
m
machinery of these three states. co
il .
m a
@ g
7 develop a program governance structure, establish
It will help in designing a robust roadmap,
a
r
is and provide support to the state institutions to achieve
monitoring and tracking mechanisms,
the end objectives. it m
u lk
-p
s a be implemented by NITI Aayog along with McKinsey & Company and IPE
SATH programrwill
i
M
Global consortium.
l k it
Pu
Parameters such as MMR, IMR, incidence of malaria will be considered for determining
potential impact.

Q Source:
http://www.thehindu.com/business/niti-aayog-unveils-sath-programme/article18956584.ece

www.insightsias.com 104
© Insights Active Learning | All rights reserved - 78168. You may not reproduce, distribute or exploit the contents in any form without
written permission by copyright owner. Copyright infringers may face civil and criminal liability
Total Marks : 200
TEST - 1 (MOCK TEST - 1) Mark Scored : 103.33

95 The decision to locate a common well in a particular area of the village can be taken by

A. Central Government only


B. State Government only
C. District bureaucracy only
D. Panchayati Raj Institutions (PRIs)

Your Answer : D
Correct Answer : D

Answer Justification :

Learning: PRIs were established with an objective of democratic decentralization.

State legislatures can devolve powers to PRIs under 73rd constitutional amendment. Such powers
can involve management of local markets, public works, sanitation, public health etc.
7
2 27
Under these provisions, a public well can be established by PRIs. 3
1 50
4 Life - I
-9
Q Source: Improvisation: Page 30: NCERT Class VI Social and Political

. com
ai l
m
7 @g
isra
k i tm
p ul
a -
isr
M
l k it
Pu

www.insightsias.com 105
© Insights Active Learning | All rights reserved - 78168. You may not reproduce, distribute or exploit the contents in any form without
written permission by copyright owner. Copyright infringers may face civil and criminal liability
Total Marks : 200
TEST - 1 (MOCK TEST - 1) Mark Scored : 103.33

96 With reference to Buddhist disciples, consider the following.


1. Sariputta was a chief female disciple of Gautama Buddha.
2. Khema, who joined the Buddhist Sangha, was one of the queens of King Bimbisara.
3. Moggallana, a disciple of Buddha was known for his psychic powers.
4. King Ajatasatru of Magadha and King Prasenajit of Kosala became Buddha’s disciples.

Select the correct answer using the codes below.


A. 1 and 2 only
B. 3 and 4 only
C. 2, 3 and 4 only
D. 1 and 3 only

Your Answer : C
Correct Answer : C

Answer Justification :
7
2 27
Justification: Statement 1: Sariputta was one of two chief male disciples of3Gautama Buddha along
with Moggallana, counterparts to the bhikkhunis Khema and Uppalavanna, 1 50 his two chief female
disciples. They were to maintain the order of monks and nuns. - 9
4
m
l .cocases where the Buddha used his
Statement 2: The conversion of Khema was one of the irare
m a
psychic powers to make a change in the heart of another.
7 @g
is ra
Statement 3: Moggallana attained enlightenment shortly after joining the Sangha. As a teacher, he
became known for his psychic powers,
i t m which he used extensively in his teaching methods.
p ulk of Kosala and Bimbisara and Ajatasatru of Magadha too accepted
Statement 4: Kings like Prasenajit
a - and became his disciples.
the doctrines of the Buddha
isr
M
it
th
Q Source: Improvisation: Page 41: TN 11 Standard History Textbook
l k
Pu

www.insightsias.com 106
© Insights Active Learning | All rights reserved - 78168. You may not reproduce, distribute or exploit the contents in any form without
written permission by copyright owner. Copyright infringers may face civil and criminal liability
Total Marks : 200
TEST - 1 (MOCK TEST - 1) Mark Scored : 103.33

97 With reference to the International Telecommunication Union (ITU), consider the following
statements.
1. It is a non-profit non-governmental organization comprising private sector ISP representatives.
2. It releases the Global Cybersecurity Index (GCI).
3. It is responsible for allocating global radio spectrum and satellite orbits.
4. It develops the technical standards that ensure networks and technologies to seamlessly
interconnect.
5. NASSCOM is the nodal agency for coordinating with ITU from India, as a member country.
6. It strives to improve access to Information and Communication Technology among the underserved
communities worldwide.

Select the correct answer using the codes below.


A. 1, 4 and 5 only
B. 2, 3, 4 and 6 only
C. 3, 5 and 6 only
D. 1, 2, 3, 4, 5 and 6
7
2 27
Your Answer : 0 3
5
Correct Answer : B
41
-9
Answer Justification :
. com
ai l
m specialised agency for information and
Justification: Statement 1: ITU is the United Nations
g
communication technologies (ICTs). ITU has 193
7 @ countries as its members and also 800 private-
sr
sector entities and academic institutions. a
m i
t
lki 23rd (“maturing category” of the index ) out of 165 nations in
Statement 2: India has been ranked
u
p Index (GCI) that measures the commitment of nations across the
the second Global Cybersecurity
-
sra
world to cyber security.
i
ITU has ranked
k itM
the countries based on the countries’ legal, technical and organisational
u l educational and research capabilities, as well as their cooperation in information-
P
institutions,
sharing networks.

Statement 3 and 4: There must be a global agency that coordinates these standards (lie ICANN),
else telecommunication between different nations would have been difficult.

Statement 6: India is a member country since 1869 (british times), but it is the Department of
Telecommunications that is the nodal agency for such coordination.

Q Source: Release of Global Cybersecurity Index (GCI) 2017

www.insightsias.com 107
© Insights Active Learning | All rights reserved - 78168. You may not reproduce, distribute or exploit the contents in any form without
written permission by copyright owner. Copyright infringers may face civil and criminal liability
Total Marks : 200
TEST - 1 (MOCK TEST - 1) Mark Scored : 103.33

98 Firoz Tughlaq’s welfare state included which of these administrative measures?


1. Treating Hindus and Muslims alike in terms of political and economic rights
2. Abolition of the practice of slavery
3. Establishment of government department to take care of orphans and widows

Select the correct answer using the codes below.


A. 1 and 3 only
B. 2 and 3 only
C. 3 only
D. 1 and 2 only

Your Answer : C
Correct Answer : C

Answer Justification :
7
27towards Shia
Justification: Statement 1: As Firoz was guided by the ulemas, he was intolerant
2
03 Jiziya. In this respect
Muslims and Sufis. He treated Hindus as second grade citizens and imposed
5
he was the precursor of Sikandar Lodi and Aurangazeb. 1 4
-9
m the defeated soldiers and young
Statement 2: Also he increased the number of slaves by capturing
o
persons. So, 2 is wrong. il. c
m a
7 @g
Statement 3: A new department called Diwan-i-Khairat was created to take care of orphans and

sra for poor Muslims were also established.


widows. Free hospitals and marriage bureaus
i
i m
tStandard
k
th
Q Source: Page 185-186: TN 11 History Textbook
u l
- p
i sra
k itM
l
Pu

www.insightsias.com 108
© Insights Active Learning | All rights reserved - 78168. You may not reproduce, distribute or exploit the contents in any form without
written permission by copyright owner. Copyright infringers may face civil and criminal liability
Total Marks : 200
TEST - 1 (MOCK TEST - 1) Mark Scored : 103.33

99 With reference to the National Mission on Education through ICT (NMEICT), consider the following:
1. Certification of attainments of any kind at any level acquired through formal or non-formal means in
conventional or non-conventional fields
2. Platform for sharing of ideas and techniques and pooling of knowledge resources
3. Scholarship and Talent management including identification, nurturing and disbursement
electronically

Which of the above is/are the objectives of NMEICT?


A. 1 and 2 only
B. 2 only
C. 1 and 3 only
D. 1, 2 and 3

Your Answer : D
Correct Answer : D
7
Answer Justification :
2 27
0 3
5
Learning: These are the guiding philosophies for NMECIT –
41
-9
(a) no talent of the country should be allowed to go waste,
. com
ai l
(b) all the services available through the content delivery portal Sakshat (NMEICT) should be free
g m
and freely available material on the web should@ used so as to avoid reinventing the wheel.
be

s r a7
m i
Scheme: It has been envisaged as a Centrally Sponsored Scheme to leverage the potential of ICT,
in teaching and learning process t
kifor the benefit of all the learners in Higher Education Institutions
u l
in any time anywhere mode.
- p
a
srmajor
The Mission has two
M i components:

l k it
u
Pproviding connectivity, along with provision for access devices, to institutions and learners;

Content generation.

The content portion of this Mission would have an ambitious vision of catering to the learning needs
of more than 50 crore Indians (working population) and of providing a one stop solution to all the
requirements of the learning community.

The objectives and details can be read at


http://mhrd.gov.in/sites/upload_files/mhrd/files/upload_document/MissionDocument.pdf

www.insightsias.com 109
© Insights Active Learning | All rights reserved - 78168. You may not reproduce, distribute or exploit the contents in any form without
written permission by copyright owner. Copyright infringers may face civil and criminal liability
Total Marks : 200
TEST - 1 (MOCK TEST - 1) Mark Scored : 103.33

100 India Infrastructure Fund (IIF)


1. is a SEBI-registered domestic venture capital fund
2. sponsored entirely by foreign governments
3. invests only in greenfield projects

Select the correct answer using the codes below.


A. 1 only
B. 2 and 3 only
C. 1 and 3 only
D. None of the above

Your Answer : D
Correct Answer : A

Answer Justification :
7
2 27
Justification: Statement 1: It is focused on long-term equity investments in a diversified portfolio of
infrastructure projects. 03
4 15
Statement 2: IIF has been sponsored by IDFC Limited (IDFC), along - 9 with Citigroup Inc.(Citi) and
India Infrastructure Finance Company Limited (IIFCL) as founder
o m investors.
c
il. of the Government of India.
m
The establishment of IIF has received the strong supporta
g
7@ greenfield, brownfield and operational
Statement 3: IIF’s portfolio is expected toacomprise
is r
assets/projects in core infrastructure sub-sectors including transport etc.
t m
u lkibillion with investor commitments from institutional investors in
- p
The current size of IIF is INR 38

sra
India, USA, Canada, Europe, Japan and the Middle-East.
i
it MInfrastructure Investment Bank (AIIB) has approved $150 million equity investment
China-led Asian
k
loan to u l India Infrastructure Fund. This loan will be the bank’s first equity investment to fund
the
P
private projects.

Q Source: As mentioned above (China AIIB investment)

www.insightsias.com 110
© Insights Active Learning | All rights reserved - 78168. You may not reproduce, distribute or exploit the contents in any form without
written permission by copyright owner. Copyright infringers may face civil and criminal liability
Total Marks : 200
TEST - 1 (MOCK TEST - 1) Mark Scored : 103.33

7
2 27
0 3
5
41
-9
. com
ai l
gm
7 @
isra
i t m
p ulk
a -
i sr
M
l k it
Pu

www.insightsias.com 111
© Insights Active Learning | All rights reserved - 78168. You may not reproduce, distribute or exploit the contents in any form without
written permission by copyright owner. Copyright infringers may face civil and criminal liability

You might also like